Download as pdf or txt
Download as pdf or txt
You are on page 1of 43

Schemes Test #2

Schemes Test #2
ANSWER KEY

Q. No. Answer Q. No. Answer Q. No. Answer Q. No. Answer Q. No. Answer
1 C 11 B 21 D 31 A 41 C
2 C 12 B 22 C 32 C 42 A
3 D 13 B 23 C 33 D 43 C
4 B 14 A 24 D 34 C 44 B
5 C 15 A 25 C 35 C 45 B
6 D 16 A 26 C 36 C 46 B
7 B 17 C 27 C 37 B 47 D
8 B 18 A 28 B 38 A 48 B
9 D 19 C 29 C 39 C 49 D
10 C 20 B 30 C 40 D 50 C

For More Study Material, Visit: studyiq.com Page no.


1
Schemes Test #2

Explanation
1. Answer: C
Explanation: Dam Health And Rehabilitation Monitoring Application (DHARMA) is a web-
based software package to support the effective collection and management of Dam Safety data in
respect of all large dams of India. The software is designed for users at Central, State and Dam level,
with user permission rights governed by their respective licences.
Source: https://damsafety.cwc.gov.in/dharma/Home1/index.php

2. Answer: C
Explanation:
● All three are correct: Galvanizing Organic Bio-Agro Resources Dhan (GOBARdhan) is a vital
umbrella initiative of Government of India, based on the whole of Government approach and aims
to convert waste to wealth towards promoting a circular economy.
○ Department of Drinking Water and Sanitation, Ministry of Jal Shakti is the coordinating
department for GOBARdhan.
○ Objectives of GOBARdhan:
■ To support villages, safely manage their cattle and agricultural waste,and make the villages
clean.
■ To support communities in converting cattle and organic waste into wealth using treatment
systems.
■ To convert organic waste, especially cattle waste, to biogas and organic manure for use in
rural areas.
■ To promote environmental sanitation and curb vector-borne diseases through effective
disposal of waste in rural areas.
■ To promote rural employment and income generation opportunities by involving
entrepreneurs, SHGs and youth groups insetting up, operation and managing GOBARdhan
units.

3. Answer: D
Explanation:
Atal Bhujal Yojana (ABHY), named in honor of former Prime Minister Atal Bihari Vajpayee, is a
scheme aimed at improving groundwater management. It was launched on 25 December 2019 under
the Jal Jeevan Mission.
● Statement 1 is incorrect: Atal Bhujal Yojana is not implemented on a pan-India basis. Instead,
it is focused on specific areas in the country that are identified as having critical and over-exploited
groundwater levels. The scheme is operational in seven states: Gujarat, Haryana, Karnataka,
Madhya Pradesh, Maharashtra, Rajasthan, and Uttar Pradesh. These states were selected based on
several criteria, including the prevalence of over-exploited and critical groundwater blocks and the
level of groundwater usage.
● Statement 2 is incorrect: The scheme is implemented solely by the Ministry of Jal Shakti. The
Ministry of Environment, Forest, and Climate Change is not jointly responsible for the
implementation of Atal Bhujal Yojana. The scheme is specifically designed to address the
challenges of groundwater depletion and promote sustainable groundwater management
practices. It involves community participation to make water management a collective
responsibility. The Ministry of Jal Shakti oversees the scheme's execution, focusing on institutional
strengthening, capacity building, and encouraging behavioral changes at the community level for
sustainable groundwater resource management.

4. Answer: B
Explanation:
● Statement 1 is incorrect: Phase 2 of the Dam Rehabilitation and Improvement Project (central
sector scheme) is co-financed, but not by the World Bank and Asian Development Bank (ADB)
together. Instead, it is co-financed by the World Bank and the Asian Infrastructure Investment

For More Study Material, Visit: studyiq.com Page no.


1
Schemes Test #2

Bank (AIIB). This collaboration aims to pool resources and expertise to address the critical need
for dam safety and improve water resource management in India.
● Statement 2 is correct: The primary goal of the DRIP is to improve the safety and operational
performance of selected existing dams across the participating states in India. This includes
implementing necessary rehabilitation measures, enhancing dam safety monitoring and
management practices, and strengthening the institutional framework and capacities of the entities
responsible for dam safety.

5. Answer: C
Explanation:
● Option (c) is correct:The e-Kranti initiative is a critical component of the Digital India campaign.
Its primary objective is to transform e-governance for transforming governance by delivering
government services electronically to citizens through integrated and interoperable systems via
multiple modes, ensuring efficiency, transparency, and reliability of such services at affordable
costs.
● This initiative encompasses various sectors such as health, education, agriculture, and more, aiming
to make all government services accessible to the common man in their locality, through common
service delivery outlets, and ensure efficiency, transparency, & reliability of services at affordable
costs.

6. Answer: D
Explanation:
● Option (d) is correct: The National Supercomputing Mission (NSM) of India aims to empower
national academic and R&D institutions spread across the country by installing a vast
supercomputing grid comprising more than 70 high-performance computing facilities. These
supercomputers are intended to facilitate research and development in various fields, significantly
enhancing India's capabilities in solving complex problems. The application areas of NSM include
Climate Modelling, Weather Prediction, Aerospace Engineering, Computational Biology,
Molecular Dynamics, and Finance, among others.

7. Answer: B
Explanation:
Pradhan Mantri Gramin Digital Saksharta Abhiyan (PMGDISHA) is a scheme launched by the
Government of India aimed at imparting digital literacy to rural citizens of the country.
● Statement 1 is Incorrect: The PMGDISHA scheme is designed to cater to the rural population
aged between 14 and 60 years.
○ This includes school children, college students, marginalised communities, farmers, artisans,
and the rural workforce in general.
● Statement 2 is Correct: The program indeed prioritises various groups within the rural population,
including non-smartphone users, who might have limited or no access to computers or the internet.
○ This initiative aims to bridge the digital divide and promote digital inclusion.
○ By focusing on non-smartphone users, the program seeks to upgrade their skills and enable
them to use digital devices for communication, accessing a range of government services,
undertaking cashless transactions, etc.

8. Answer: B
Explanation:
● Statement 1 and 2 are Correct: The STP Scheme is a 100% export-oriented scheme for the
development and export of computer software, including export of professional services using
communication links or physical media. As a unique scheme, it focuses on one sector, i.e. computer
software.
● Statement 3 is Incorrect: 100% foreign equity is permitted.
● Statement 4 is Correct: All the imports of Hardware & Software in the STP units are completely
duty free, import of second-hand capital goods are also permitted.

For More Study Material, Visit: studyiq.com Page no.


2
Schemes Test #2

9. Answer: D
Explanation
● Option (d) is correct: 'MeghRaj' is the Government of India's initiative, launched to leverage
cloud computing technology for efficient delivery of government services to the citizens. The
objective of 'MeghRaj' is multifaceted, focusing primarily on enhancing the efficiency,
flexibility, and scalability of e-governance services while ensuring optimal utilization of
Information and Communication Technology (ICT) infrastructure and reducing costs.
● The National Informatics Centre (NIC) is providing National Cloud services under the initiative
MeghRaj. The services offered are as follows.
○ Infrastructure as a Service (IaaS): IaaS provides you basic virtual compute infrastructure
resources like CPU, Memory, Disk Storage attached to blank VMs with allowing you to install
OS, using ISOs, from scratch and customization. However you have to use your own licenses
for OS and Application software (if any).
● Platform as a Services (PaaS): PaaS provides pre-installed web and database servers so that you
can publish and run web application without worrying about server setup. The servers are pre
configured ready with basic security hardening. Use PaaS service to quickly deploy servers and
publish your web applications. The OS & Application Software licenses are provided by us as part
of offering.
● Software as a Services (SaaS): This provides on demand software service. SaaS is a software
delivery model where users are not responsible for supporting the application or any of the
components. The server infrastructure, OS and software is being managed by cloud services. If you
are having web application and want to distribute it to users, use our Cloud Service to deliver
through Software as a Service.
● Storage as a Service (STaaS): STaaS provides need based storage solution . It provides excellent
alternative to the traditional on-site and dedicated storage systems and reduces the complexities of
deploying and managing multiple storage tiers. You can use it to mitigate risks in disaster recovery,
provide long-term retention for records and enhance both continuity and availability.
● Hosting Environments: NIC Cloud Services provides 3 different types of environment for creating
virtual machines i.e. Production, Staging and Development so that you keep your VM segregated
and manage them properly based on the business need for both PaaS as well as IaaS service model.

10. Answer: C
Explanation:
The nine pillars of digital India includes:
● Broadband Highways
● Universal Access to Mobile Connectivity
● Public Internet Access Programme
● e-Governance: Reforming Government through Technology
● e-Kranti - Electronic Delivery of Services
● Information for All
● Electronics Manufacturing
● IT for Jobs
● Early Harvest Programmes.
Source: https://digitalindia.gov.in/programme-pillars/

11. Answer: B
Explanation:
● Statement 1 is Incorrect: The SECURE Himalaya project is a part of “Global Partnership on
Wildlife Conservation and Crime Prevention for Sustainable Development” (Global Wildlife
Program).
○ It was launched in 2017 as a joint project between the Ministry of Environment, Forest and
Climate Change (MOEFCC) and the United Nations Development Programme (UNDP) and
funded by the Global Environment Facility (GEF).
● Statement 2 is Correct: Objectives:

For More Study Material, Visit: studyiq.com Page no.


3
Schemes Test #2

○ Conservation of key biodiversity areas and their effective management to secure long-term
ecosystem resilience, habitat connectivity and conservation of snow leopard and other
endangered species and their habitats
○ Securing sustainable community livelihoods and natural resource management in high range
Himalayan ecosystems
○ Enhancing enforcement, monitoring and cooperation to reduce wildlife crime and related
threats
○ Knowledge, advocacy, communication and information systems established.

12. Answer: B
Explanation:
● The National Action Plan on Climate Change (NAPCC) was launched on 30th June 2008.
● It outlines a national strategy that aims to enable the country to adapt to climate change and enhance
the ecological sustainability of India‘s development path.
● There are eight National Missions which are part of NAPCC:
1. National Solar Mission
2. National Mission for Enhanced Energy Efficiency
3. National Mission on Sustainable Habitat
4. National Water Mission
5. National Mission for Sustaining the Himalayan Ecosystem
6. National Mission for a Green India
7. National Mission for Sustainable Agriculture
8. National Mission on Strategic Knowledge for Climate Change

13. Answer: B
Explanation:
● Statement 1 is Incorrect: The Leadership Group for Industry Transition (LeadIT) gathers countries
and companies that are committed to action to achieve the Paris Agreement.
○ It was launched by the governments of Sweden and India at the UN Climate Action Summit in
September 2019.
● Statement 2 and 3 are Correct: The LeadIT (Leadership for Industry Transition) initiative focuses
on low carbon transition of the hard to abate industrial sector and is supported by the World
Economic Forum.

14. Answer: A
Explanation:
● Statement 1 and 3 are incorrect: PM Formalisation of Micro Food Processing Enterprises (PM-
FME) Scheme is a Central Sponsored Scheme which aims to address the challenges faced by the
micro-enterprises and to tap the potential of groups and cooperatives in supporting the upgradation
and formalisation of these enterprises. About 2 lakh Micro food processing enterprises will be given
direct assistance in the form of credit linked subsidy.
○ The scheme envisages an outlay of ₹ 10,000 crores over a period of five years from 2020-21 to
2024-25. The expenditure under the scheme would be shared in a 60:40 ratio between Central
and State Governments, in a 90:10 ratio with the North
○ In Eastern and the Himalayan States, a 60:40 ratio with UTs with the legislature and 100% by
the Center for other UTs
● Statement 2 is correct: Features of the PM-FME Scheme are:
○ One District One Product:The scheme adopts the component of ODOP to reap the benefit
of scale in terms of procurement of inputs, availing common services and marketing of
products.
■ The state government would identify one product for a district keeping in view the
existing clusters and availability of raw materials.
○ Priority sectors: The scheme focuses on waste to wealth products, minor forest products and
Aspirational Districts.

For More Study Material, Visit: studyiq.com Page no.


4
Schemes Test #2

○ Nodal Bank- Union Bank of India.


Source: https://pib.gov.in/PressReleasePage.aspx?PRID=1984009

15. Answer: A
Explanation:
● Statement 1 is correct: The PMMSY is an umbrella scheme with two separate components namely
(a) Central Sector Scheme (CS) and (b) Centrally Sponsored Scheme (CSS). The Centrally
Sponsored Scheme (CSS) Component is further segregated into Non-beneficiary oriented and
beneficiary orientated sub-components/activities under the following three broad heads: (i)
Enhancement of Production and Productivity (ii) Infrastructure and Post-Harvest Management (iii)
Fisheries Management and Regulatory Framework
● Statement 2 is incorrect: The scheme is to be implemented in all states/UTs across the country
(both coastal and landlocked).
● Statement 3 is correct: Objectives of PMMSY
○ 1. Harness the potential of the fisheries sector in a sustainable, responsible, inclusive, and
equitable manner.
○ 2. Enhance fish production and productivity through expansion, intensification, diversification,
and productive utilization of land and water.
○ 3. Modernize and strengthen the value chain including post-harvest management and quality
improvement.
○ 4. Double fishers and fish farmers’ incomes and generate meaningful employment.
○ 5. Enhance the contribution of the fisheries sector to Agricultural GVA and exports.
○ 6. Ensure social, physical, and economic security for fishers and fish farmers.
○ 7. Build a robust fisheries management and regulatory framework.
Source: https://static.pib.gov.in/WriteReadData/userfiles/PMMSY%20BookEnglish.pdf

16. Answer: A
Explanation:
● Statement 1 is correct: Raising and Accelerating MSME Performance Programme–(RAMP)
is a Central Sector Scheme and aims to scale up the implementation capacity and coverage of
MSMEs in the States, with impact enhancement of existing MSME schemes.
● Statement 2 is correct: Assistance for this scheme is being provided by the World Bank.
● Components of the Scheme:
o Disbursement linked indicators(DLI)-The Finds from World Bank will be provided on
fulfilling several indicators like reducing incidence of delayed payments, better collaboration
between centre-states in MSME acceleration, enhancing the effectiveness of MSME Schemes
etc.
o Strategic Investment Plans (SIPs)- A significant component of RAMP scheme is preparation
of SIPs. It includes a plan for mobilising MSMEs, identification of key constraints, set
milestones for priority sector intervention etc.
● Statement 3 is incorrect: To be eligible for the RAMP scheme, MSME should be registered under
the Micro, Small and Medium Enterprises Development (MSMED) Act, 2006, and have a valid
Udyog Aadhaar Number (UAN). MSME should also have a good track record and credit rating.
Source: https://pib.gov.in/PressReleasePage.aspx?PRID=1811360

17. Answer: C
Explanation:
● Statement 1 is correct and Statement 2 is incorrect: The purpose of National Green Hydrogen
Missions is to make India energy independent by 2047 and achieve Net Zero by 2070.
● Other objectives:
○ To incentivise the commercial production of green hydrogen and make India a net exporter
of the fuel
○ To decarbonise the energy sector and Reduce dependence on imported fossil fuels
○ To facilitate demand creation, production and utilisation of Green Hydrogen

For More Study Material, Visit: studyiq.com Page no.


5
Schemes Test #2

○ To create export opportunities for Green Hydrogen and its derivatives


○ Development of indigenous manufacturing capabilities
● Statement 3 is correct: Oil India Limited (OIL) has taken the first significant step towards Green
Hydrogen Economy in India with the commissioning of India’s First 99.999% pure Green
Hydrogen pilot plant, with an installed capacity of 10 kg per day at its Jorhat Pump Station in
Assam. The plant was commissioned in a record time of 3 months.
Source: https://pib.gov.in/PressReleasePage.aspx?PRID=1888547

18. Answer: A
Explanation:
● Statement 1 is correct: Development of solar cities scheme is an initiative to promote the use of
renewable energy in urban areas by providing support to municipal corporations to develop Solar
Cities. It aims at providing financial assistance up to ₹ 50 lakhs/ city/ town.
○ Criteria for identification of cities-
○ A city with population between 50,000 to 50 lakh \
○ Promotes energy efficiency and renewable energy
● Statement 2 is incorrect: The Ministry of New and Renewable Energy is the nodal Ministry
for implementing this scheme. A total of 60 cities are approved to be supported for development
as solar cities.
● Statement 3 is incorrect: Sanchi in Madhya Pradesh is the first solar city of India.
○ Modhera in Gujarat is the first solar village of the country.
Source: https://pib.gov.in/Pressreleaseshare.aspx?PRID=1519372
https://newsonair.gov.in/Main-News-Details.aspx?id=467334

19. Answer: C
Explanation:
● Statement 1 is incorrect while Statements 2, 3 and 4 are correct: The Government of India
implements Anemia Mukt Bharat (AMB) strategy under POSHAN Abhiyaan with the target for
reducing anemia among Indian people. The Anemia Mukt Bharat strategy has been designed to
reduce the prevalence of anemia by 3% points per year among children, adolescents and women in
the reproductive age group (15–49 years), between the years 2018 and 2022. Government aims to
provide preventive and curative mechanisms through a 6X6X6 strategy including six target
beneficiaries, six interventions and six institutional mechanisms for all stakeholders to implement
the strategy.
Major Interventions by Government under AMB Strategy include:
o Prophylactic Iron and Folic Acid Supplementation.
o Intensified year-round Behaviour Change Communication (BCC) Campaign and delayed cord
clamping
o Periodic deworming of children and adolescents.
o Testing of anemia using digital methods and point of care treatment.
o Addressing non-nutritional causes of anemia in endemic pockets with special focus on malaria,
hemoglobinopathies and fluorosis.
o Convergence and coordination with line departments and other ministries.
o Engaging National Centre of Excellence and Advanced research on Anemia Control for
capacity building of health care providers.
o Monitoring progress in States/UTs using Anemia Mukt Bharat Dashboard.
Source: UPSC PYQ 2023

20. Answer: B
Explanation:
● Statement 1 is incorrect: Janani Suraksha Yojana (JSY) is a safe motherhood intervention under
the National Rural Health Mission (NHM). The Janani Suraksha Yojana was launched in April
2005 by modifying the National Maternity Benefit Scheme (NMBS).

For More Study Material, Visit: studyiq.com Page no.


6
Schemes Test #2

● Statements 2 and 3 are correct: Janani Suraksha Yojana (JSY) is being implemented with the
objective of reducing maternal and infant mortality by promoting institutional delivery among
pregnant women. The scheme is under implementation in all states and Union Territories (UTs),
with a special focus on Low Performing States. JSY is a centrally sponsored scheme, which
integrates cash assistance with delivery and post-delivery care. The Yojana has identified
Accredited Social Health Activist (ASHA) as an effective link between the government and
pregnant women.
● Statement 4 is incorrect: The scheme focuses on poor pregnant woman with a special dispensation
for states that have low institutional delivery rates, namely, the states of Uttar Pradesh, Uttarakhand,
Bihar, Jharkhand, Madhya Pradesh, Chhattisgarh, Assam, Rajasthan, Orissa, and Jammu and
Kashmir. The scheme also provides performance-based incentives to women health volunteers
known as ASHA (Accredited Social Health Activist) for promoting institutional delivery among
pregnant women. Under this initiative, eligible pregnant women are entitled to get JSY benefits
directly into their bank accounts.
Source: UPSC PYQ 2023

21. Answer: D
Explanation:
Statement 1 is correct: The Members of Parliament Local Area Development Scheme (MPLADS) is
an ongoing The MPLADS is a Central Sector Scheme fully funded by the Government of India and it
was briefly suspended during the pandemic course. The Scheme enables the Members of Parliament
to recommend works for the creation of durable community assets in their constituencies namely
drinking water, education, public health, sanitation, roads etc. Individual benefits and movable
& non-durable assets (certain exemptions excepted) are generally not allowed under the scheme.
Statement 2 is correct: The scheme puts special focus on the development of areas with Scheduled
Caste and Scheduled Tribe populations. MPs are to recommend works costing at least 15% of the
annual MPLADS entitlement for areas inhabited by SC populations and 7.5% for areas with ST
populations.
Statement 3 is incorrect: Each MP is entitled to ₹5 crores annually to spend on projects for community
use in his or her constituencies. The unspent amount is kept in a non-lapsable account with the
district authority in the constituency and can be used in subsequent years.
Statement 4 is correct: MPs do not directly receive funds under MPLADS. The Centre directly
transfers the sanctioned amount in two instalments of Rs. 2.5 crore to the district authorities of the
concerned MP’s nodal district after a recommended project gets approval. The District Authority
would be responsible for overall coordination and supervision of the works under the scheme at
the district level and inspect at least 10% of the works under implementation every year. The
District Authority should involve the MPs in the inspections of projects to the extent feasible.
Source: UPSC PYQ 2020

22. Answer: C
Explanation:
Statement 1 is incorrect: Pradhan Mantri Kaushal Vikas Yojana (PMKVY) is the flagship scheme of
the Ministry of Skill Development & Entrepreneurship (MSDE) implemented by National Skill
Development Corporation.
Statement 2 is correct: It aims to impart short-duration skill development training, training in soft
skills, entrepreneurship, financial, digital literacy and certification to youth and to make them
employable for better livelihood across the country.
Statement 3 is correct: The objective of this Skill Certification Scheme is to enable a large number of
Indian youth to take up industry-relevant skill training that will help them in securing a better
livelihood. It aims to align the competencies of the un-regulated workforce of the country, to the
National Skill Qualification Framework (NSQF).
Source: UPSC PYQ 2018

For More Study Material, Visit: studyiq.com Page no.


7
Schemes Test #2

23. Answer: C
Explanation:
Option (c) is correct: Integrated Watershed Development Program is implemented by the Department
of Land Resources of the Ministry of Rural Development.
The main aims of this programme are as follows:
● To restore the ecological balance by harnessing, conserving and developing degraded natural
resources such as soil, vegetative cover and water.
● Prevention of soil run-off, soil loss.
● Regeneration of natural vegetation.
● Rainwater harvesting and recharging of the ground Water table.
● Enabling multi-cropping and the introduction of diverse agro-based activities, which help to provide
sustainable livelihoods to the people residing in the watershed area.
Linking the country's perennial rivers with seasonal rivers is not the component and benefit of
the Integrated Watershed Development Programme.
Source: UPSC PYQ 2014

24. Answer: D
Explanation:
Statement 1 is incorrect: The project is being implemented by the Directorate General of
Employment, Ministry of Labour & Employment. The Union Ministry of Labour and Employment
has also started free online career skills training through its National Career Service project for job-
seekers registered with it.
Statement 2 is incorrect: It works towards bridging the gap between job seekers (Educated) and
employers, candidates seeking training and career guidance, and agencies providing training and career
counseling. The aim is to provide a variety of employment-related services like job matching,
career counseling, vocational guidance, information on skill development courses, internships,
etc. All the services are free of cost for all stakeholders including jobseekers, employers, training
providers, and placement organizations.

25. Answer: C
Explanation:
● Statement 1 is correct: The Unnat Jyoti by Affordable LEDs for All (UJALA) scheme, launched
by the Government of India, primarily focuses on promoting energy efficiency and conservation
by distributing LED bulbs at subsidized rates. The initiative is aimed at replacing incandescent
bulbs with LEDs, which consume significantly less electricity and have a longer lifespan, thereby
reducing the overall energy consumption in households and commercial establishments across the
country.
● Statement 2 is incorrect: the primary objective of the UJALA scheme is not to enhance the
production of solar panels. While promoting renewable energy sources, including solar energy, is
a part of India's broader energy policy, the UJALA scheme specifically targets energy efficiency
through the adoption of LED lighting technology. The focus is on reducing energy demand and the
environmental impact of energy consumption rather than directly influencing the solar panel
production industry.

26. Answer: C
Explanation:
● Statement 1 is correct: Atal Mission for Rejuvenation and Urban Transformation (AMRUT) was
launched in 2015 in selected 500 cities and towns across the country. The Mission focuses on
development of basic infrastructure, in the selected cities and towns, in the sectors of water
supply; sewerage and septage management; storm water drainage; green spaces and parks;
and non-motorized urban transport. A set of Urban Reforms and Capacity Building have been
included in the Mission. AMRUT Mission has been subsumed under AMRUT 2.0, which was
launched on 01st October, 2021. Mission also has a reform agenda on ease of living of citizens

For More Study Material, Visit: studyiq.com Page no.


8
Schemes Test #2

through reduction of non-revenue water, recycle of treated used water, rejuvenation of water bodies,
augmenting double entry accounting system, urban planning, strengthening urban finance etc.
● Statement 2 is incorrect: The Ministry of Urban Development, Government of India, launched
the Heritage City Development and Augmentation Yojana (HRIDAY) scheme, with a focus
on holistic development of heritage cities. The main objective of HRIDAY is to preserve character
of the soul of heritage city and facilitate inclusive heritage linked urban development by exploring
various avenues including involving private sector. HRIDAY is a central sector scheme, where
100% funding will be provided by Government of India.
● Statement 3 is correct: The Government of India launched the PRASAD scheme in the year
2014-2015 under the Ministry of Tourism. The full form of the PRASAD scheme is ‘Pilgrimage
Rejuvenation And Spiritual Augmentation Drive’. This scheme focuses on developing and
identifying pilgrimage sites across India for enriching the religious tourism experience. It
aims to integrate pilgrimage destinations in a prioritised, planned and sustainable manner to
provide a complete religious tourism experience. The PRASAD scheme aims at paving the way
for the development and promotion of religious tourism in India.
Source: https://pib.gov.in/PressReleasePage.aspx?PRID=1885837 https://tourism.gov.in/schemes-
and-guidelines/schemes/scheme-guidelines-
prasad#:~:text=The%20full%20form%20of%20the,enriching%20the%20religious%20tourism%20ex
perience. https://vikaspedia.in/social-welfare/urban-poverty-alleviation-1/hriday-scheme-and-
guidelines

27. Answer: C
Explanation:
● Statements 1 and 3 are correct while statement 2 is incorrect: Indian Railways have prepared a
National Rail Plan (NRP) for India – 2030. The Plan is to create a ‘future ready’ Railway system
by 2030. The key objectives of the National Rail Plan are: -
o Formulate strategies based on both operational capacities and commercial policy initiatives to
increase modal share of the Railways in freight to 45%.
o Reduce transit time of freight substantially by increasing average speed of freight trains
to 50Kmph.
o Identify new Dedicated Freight Corridors.
o Identify new High Speed Rail Corridors.
o Assess rolling stock requirement for passenger traffic as well as wagon requirement for freight.
o Assess Locomotive requirement to meet twin objectives of 100% electrification (Green
Energy) and increasing freight modal share.
o Assess the total investment in capital that would be required along with a periodical break up.
o Sustained involvement of the Private Sector in areas like operations and ownership of rolling
stock, development of freight and passenger terminals, development/operations of track
infrastructure etc.
Source: https://pib.gov.in/PressReleasePage.aspx?PRID=1806617

28. Answer: B
Explanation:
● Option (a) is correct: The Cabinet Committee on Economic Affairs recently approved the proposal
of the Ministry of Information and Broadcasting regarding Central Sector Scheme “Broadcasting
Infrastructure and Network Development” (BIND) for the infrastructure development of
Prasar Bharati i.e. All India Radio (AIR) and Doordarshan (DD). The “Broadcasting
Infrastructure and Network Development” scheme of the Ministry is the vehicle for providing
financial support to Prasar Bharati for expenses related to expansion and upgradation of its
broadcasting infrastructure, content development and civil work related to the organization.
● Option (b) is incorrect and Option (c) is correct: The BIND scheme will enable the public
broadcaster to undertake a major upgradation of its facilities with better infrastructure which will
widen its reach, including in the Left Wing Extremism (LWE), border and strategic areas and
provide high quality content to the viewers. Another major priority area of the scheme is the

For More Study Material, Visit: studyiq.com Page no.


9
Schemes Test #2

development of high-quality content for both domestic and international audience and
ensuring availability of diverse content to the viewers by upgradation of capacity of DTH platform
to accommodate more channels. Purchase of outside broadcasting (OB) vans and digital
upgradation of DD and AIR Studios to make them HD ready will also be done as part of the
project.
● Option (d) is correct: At present, Doordarshan operates 36 TV channels including 28 regional
channels and All India Radio operates more than 500 broadcasting centres. The Scheme will
increase coverage of AIR FM transmitters in the country to 66% by geographical area and 80% by
population up from 59% and 68% respectively. The Scheme also envisages free distribution of
over 8 lakh DD Free Dish Set Top Boxes (STBs) to people living in remote, tribal, LWE and
border areas.
Source: https://www.pib.gov.in/PressReleasePage.aspx?PRID=1888540

29. Answer: C
Explanation:
Air sports, as the names suggests, encompasses various sports activities involving the medium of air.
These include sports like air-racing, aerobatics, aero modelling, hang gliding, paragliding, para
motoring and skydiving etc.
● Statement 1 is incorrect: National Air Sport Policy 2022 (NASP 2022) has been launched by
the Ministry of Civil Aviation.
● Statement 2 is incorrect: NASP 2022 lays out the vision of making India as one of the top
sports nations by 2030, by providing a safe, affordable, accessible, enjoyable and sustainable air
sports ecosystem in India.
● Statement 3 is correct: Under the new policy, there will be four-tier governance structure for air
sports in India namely
o Air Sports Federation of India (ASFI) as the apex governing body
o National associations for individual air sports or a set of air sports, as appropriate
o Regional (e.g. West/ South/ Northeast etc.) or State and Union Territory level units of the
national air sports associations, as appropriate; and
o District-level air sports associations, as appropriate.
● Statement 4 is correct: Key Objectives of NASP 2022:
o Promote an air sports culture in the country
o Enable adoption of international good practices in safety including but not limited to, air sports
infrastructure, equipment, operations, maintenance and training
o Develop a simple, stakeholder-friendly and effective governance structure
o Enhance participation and success of Indian sportspersons in global air sports events; and
o Promote design, development and manufacturing of air sports equipment in India in line
with the Atmanirbhar Bharat Abhiyan.
Source: https://pib.gov.in/PressReleasePage.aspx?PRID=1831844

30. Answer: C
Explanation:
The "Sustainable Aquaculture In Mangrove Ecosystems" (SAIME) project aims to promote the
sustainable design of aquaculture systems by focusing on protecting mangroves and improving local
living conditions. Global Nature Fund (project coordinator) and Naturland e.V. are working together in
a consortium and in close cooperation with two selected local project partners. The project is funded by
the German Federal Ministry for Economic Cooperation and Development (BMZ).
● Option (c) is correct: A large proportion of the project measures is being implemented in the
Sundarbans, where the world’s largest mangrove forest stretches over an area of 10,000 km² in the
federal state of West Bengal in India and the Khulna Division in Bangladesh. Under the initiative,
Sustainable Aquaculture In Mangrove Ecosystem (SAIME), farmers have taken up
cultivation of shrimp in ponds in and around the mangrove trees at Chaital in West Bengal’s
North 24 Parganas, and 10 hectares at Madhabpur in adjoining South 24 Parganas. The two
local project partners are the Bangladesh Environment and Development Society (BEDS) and the

For More Study Material, Visit: studyiq.com Page no.


10
Schemes Test #2

Nature Environment & Wildlife Society of India (NEWS). The project’s agenda is to establish
robust co-operation and communication links.
Source:https://www.naturland.de/en/producers/projects/international-commitment/sustainable-
aquaculture-in-mangrove.html

31. Answer: A
Explanation:
● Statement 1 is incorrect: The Ministry of Housing & Urban Affairs launched a scheme PM Street
Vendor's AtmaNirbhar Nidhi (PM SVANidhi) to empower Street Vendors (does not mention
Traditional Artisans at all) by not only extending loans to them, but also for their holistic
development and economic upliftment. The scheme intends to facilitate collateral free working
capital loans of up to INR 10,000/- of one-year tenure, to approximately 50 lakh street vendors, to
help resume their businesses in the urban areas, including surrounding peri-urban/rural areas.
● Statement 2 is correct: The PM SVANidhi scheme offers incentives in the form of:
○ interest subsidy @ 7% per annum on regular repayment of loan
○ cashback up to INR1200/- per annum on undertaking prescribed digital transactions
○ eligibility for enhanced next tranche of loans
● Statement 3 is incorrect: The Ministry of Housing and Urban Affairs (MoHUA) recently launched
the ‘SVANidhi se Samriddhi’ program in 126 cities across 14 States/ UTs, in the presence of senior
officials of States/UTs and various Central Ministries. Under the program, socio-economic
profiling of PM SVANidhi beneficiaries and their families is conducted to assess their
eligibility for 8 Government of India’s welfare schemes and facilitate sanctions of eligible
schemes. These schemes include:
○ Pradhan Mantri Jeevan Jyoti Bima Yojana,
○ PM Suraksha Bima Yojana,
○ Pradhan Mantri Jan Dhan Yojana,
○ Registration under Building and other Constructions Workers (Regulation of Employment and
Conditions of Service) Act (BOCW),
○ Pradhan Mantri Shram Yogi Maandhan Yojana,
○ National Food Security Act (NFSA) portability benefit – One Nation One Ration Card
(ONORC),
○ Janani Suraksha Yojana, and
○ Pradhan Mantri Matru Vandana Yojana (PMMVY).
Quality Council of India (QCI) is the implementing partner for the programme.
Sources:
https://pib.gov.in/PressReleaseIframePage.aspx?PRID=1816139
https://www.india.gov.in/spotlight/pm-street-vendors-atmanirbhar-nidhi-pm-svanidhi

32. Answer: C
Explanation:
● Statement 1 is correct: UDAY (Ujwal DISCOM Assurance Yojana), a Scheme for the Financial
Turnaround of Power Distribution Companies (DISCOMs) aims to improve the operational and
financial efficiency of the State DISCOMs.
● Statement 2 is correct: This scheme shall apply only to State-owned DISCOMs. DISCOM for the
purpose of this Scheme may include combined Generation, transmission and distribution
Undertakings.
● Statement 3 is correct: Objectives of the scheme are:
○ Reduce the debt burden of DISCOMs.
○ Improve the operational efficiency of DISCOMs.
○ Ensure a sustainable power supply to consumers.
○ Promote the use of renewable energy.
○ Enforcing financial discipline on DISCOMs by aligning their finances with those of the State.

For More Study Material, Visit: studyiq.com Page no.


11
Schemes Test #2

33. Answer: D
Explanation:
● Statement 1 and 2 are incorrect: The Ministry of Rural Development (MoRD) has announced the
launch of “Prajjwala Challenge” for bringing about “Transformation of the Indian Rural
Economy” with special focus on rural women belonging to the marginalised rural communities
in India. It seeks to expand interest and partnership, mobilise and harvest the power of experts,
academia, youth, start- ups, Self Help Groups (SHGs), etc. to bring new and innovative ideas to
transform the rural economy. This is a first of its kind nationwide initiative to provide thoughtful
minds a platform to present new, innovative and scalable pathways for rural economic
transformation in India. The Goal is to support 5 Innovative Ideas / Models to bring about
Transformation of the Rural Economy in India. It is not launched under PM jan dhan yojana
rather it is launched by Ministry of Rural Development.
Source: https://www.prajjwalachallenge.com/

34. Answer: C
Explanation:
● Statement 1 is correct: Street Lighting National Programme (SLNP) has been launched by the
Government of India on 5 January, 2015, with focus to replace conventional street lights with smart
and energy efficient Light Emitting Diode (LED) street lights.
○ SLNP is being implemented by Energy Efficiency Services Limited (EESL), a joint venture
company of Public Sector Undertakings (PSUs) under Ministry of Power. Under this
programme, till date, over 64 lakh LED street lights have been installed, covering 28
States/Union Territories.
● Statement 2 is correct: No financial allocation is made for SLNP because the programme is run
without any budgetary allocation from Government of India.

35. Answer: C
Explanation:
Statements 1, 2 and 4 are correct: It is mandatory for the manufacturer to put the information like
List of the ingredients including additives, Nutrition information and Vegetarian/ Non-
Vegetarian on the main label, as per the Food Safety and Standards (Packaging and Labelling)
Regulations, 2011.
Statement 3 is incorrect: Recommendations, if any, made by the medical profession about the
possibility of any allergic reactions are not mandatory.
Source: UPSC CSE Prelims 2016

36. Answer: C
Explanation:
To address urban housing shortage (validated demand of 1.12 crore houses) among the EWS/ LIG and
MIG category, including the slum dwellers by ensuring a pucca house to eligible urban households by
the year 2022. (Now, the scheme has been extended till 2024)
● Statement 1 is correct: Eligibility conditions under PMAY (Urban) includes the following:
● The beneficiary family should not own a pucca house either in his/her name or in the name of
any member of his/her family in any part of India.
● Annual income cap
○ For EWS (Economically Weaker Section)- up to Rs 3 lakh
○ For LIG (Low Income Group)-Rs 3-6 lakh
○ For MIG (Middle Income Group)- Rs 6-18 lakh
● Statement 2 is correct: The beneficiaries can take advantage under one component only.
○ EWS eligible for all components.
○ LIG and MIG available only for CLSS.
○ The houses constructed/acquired with central assistance under the mission should be in the
name of the female head of the household or in the joint name of the male head of the

For More Study Material, Visit: studyiq.com Page no.


12
Schemes Test #2

household and his wife, and only in cases when there is no adult female member in the
family, the house can be in the name of male member of the household.
● Statement 3 is correct: Government has also sanctioned ‘infrastructure status’ for the affordable
housing sector, giving a boost to PMAY

37. Answer: B
Explanation
● Option (a) is incorrect: Pradhan Mantri Surakshit Matritva Abhiyan (PMSMA) was launched
in 2016. This programme aims to provide assured, comprehensive and quality antenatal care, free
of cost, universally to all pregnant women on the ninth day of every month. PMSMA guarantees
a minimum package of antenatal care services to women in their second or third trimesters
of pregnancy at designated government health facilities.
● Option (b) is correct: SUMAN Yojana or Surakshit Matritva Aashwasan Yojana is a maternity
benefit initiative launched by the Ministry of Union Health and Family Welfare. This program
provides affordable and quality healthcare solutions to pregnant women and newborns. Under
this scheme, pregnant women, sick newborns, and mothers receive zero expense access up to six
months after delivery.
● Option (c) is incorrect: LaQshya- Labour, quality improvement initiative, is a scheme launched
by the Union Ministry of Health and Family welfare, to improve the quality of care given in
the labour room and maternity operation theatres in Government Medical College Hospitals,
District Hospitals, Sub-District Hospitals, and other essential health facilities.
● Option (d) is incorrect: Pradhan Mantri Matru Vandana Yojana - A maternity benefits
program by the Ministry of Women and Child Development in which cash incentive of ₹ 5000/-
to Pregnant Women of 19 years of age or above for the first live birth. The incentive is provided
in three installments and is to be claimed within 150 days, 180 days, and at childbirth respectively.
Sources:
India Yearbook - Chapter 14: Health and Family Welfare.
https://hrex.org/laqshya-scheme#:~:text=LaQshya-
%20Labour%2C%20quality%20improvement%20initiative%2C%20is%20a%20scheme,Hospitals%2
C%20Sub-District%20Hospitals%2C%20and%20other%20essential%20health%20facilities.
https://www.myscheme.gov.in/schemes/pmmvy
Surakshit Matritva Aashwasan Yojana (myscheme.gov.in)

38. Answer: A
Explanation
● Pair 1 is incorrect: Atal Pension Yojana (APY), a pension scheme for citizens of India is focused
on the unorganized sector workers. Under the APY, guaranteed minimum pension of Rs. 1,000/-
or 2,000/- or 3,000/- or 4,000 or 5,000/- per month will be given at the age of 60 years depending
on the contributions by the subscribers. Income taxpayers are not eligible to register for APY.
● Pair 2 is incorrect: Pradhan Mantri Jeevan Jyoti Bima Yojana (PMJJBY) is an insurance
scheme by Ministry of Finance, offering life insurance cover for death due to any reason. Risk
coverage under this scheme is for Rs. 2 Lakh in case of death of the insured, due to any reason.
PM Kisan is a Central Sector scheme with 100% funding from Government Under the scheme
an income support of 6,000/- per year in three equal installments will be provided to all land holding
farmer families.
● Pair 3 is correct: Pradhan Mantri Suraksha Bima Yojana - An Accident Insurance Scheme
offering accidental death and disability cover for death or disability on account of an accident. The
premium of Rs.12/- per annum per member, will be deducted from the account holder’s bank
account through ‘auto debit’ facility
● Pair 4 is incorrect: New Pension Scheme is another retirement scheme in which the beneficiaries
will be able to withdraw 60% of the amount invested after retirement. It was introduced in the
year 2004 by the Government of India. The remaining 40% needs to be invested in annuities in
order to receive a monthly pension. 60% of the NPS Corpus is tax-free while the remaining 40%

For More Study Material, Visit: studyiq.com Page no.


13
Schemes Test #2

is taxable. Old Pension Scheme offers pensions to government employees on the basis of their last
drawn salary. The employee get 50% of the last drawn salary as a pension. No tax on pension.
Sources:
https://www.india.gov.in/spotlight/atal-pension-yojana
https://www.myscheme.gov.in/schemes/pmjjby
https://www.myscheme.gov.in/schemes/pmsby
https://www.jagranjosh.com/general-knowledge/differences-between-old-pension-scheme-and-new-
pension-scheme-1663933333-1

39. Answer: C
Explanation:
● Item 1 is correct: Going Online as Leaders (GOAL) Program was launched by Facebook in
partnership with the Ministry of Tribal Affairs (MoTA) on 15th May 2020 to digitally skill and
empower 5000 youth from tribal communities over the next five years. The project aims to
identify and mobilize 2500 renowned people from the industry which include policy makers and
influencers, teachers, artists, entrepreneurs and social workers known for their achievements in their
respective domains, to personally mentor tribal youth across India.
● Item 2 is incorrect: Sankalp Se Siddhi Scheme is an integrated yojana that the central government
has taken for the betterment of the nation. Under this scheme major social and economic issues will
be covered. It will focus on 6-7 major sectors or issues in the country. These issues will be raised
through various events under the scheme. The issues are Clean India, Literate India, Poverty-
free, Corruption-free, Terrorism-free, Communalism-free and Caste-discrimination free
India.
● Item 3 is correct: The Scheme for Minimum Support Price (MSP) for Minor Forest Produce
(MFP) and development of value chain was started by the Ministry of Tribal Affairs (MoTA) in
the FY 2013-14 with an objective of providing a fair price to MFP gatherers, enhancing their income
level and ensuring sustainable harvesting of MFPs. The objective of the MSP for MFP scheme is
to establish a framework for ensuring fair prices for the tribal gatherers, primary processing,
storage, transportation etc. while ensuring the sustainability of the resource base.
● Item 4 is correct: The Van Dhan Yojana or Van Dhan Scheme, a component of the ‘Mechanism
for Marketing of Minor Forest Produce (MFP) through Minimum Support Price (MSP) &
Development of Value Chain for MFP’ was launched on 14th April, 2018. Implemented by Tribal
Cooperative Marketing Development Federation of India (TRIFED) as the nodal agency at the
national level, the Van Dhan start-ups is a well thought master plan for the socio-economic
development of the tribal population of the country.
Sources:
https://pmjandhanyojana.co.in/sankalp-se-siddhi-scheme
https://trifed.tribal.gov.in/program/non-timber
https://trifed.tribal.gov.in/iipa
https://trifed.tribal.gov.in/pmvdy

40. Answer: D
Explanation
● MSME Samadhaan is an online portal of the Ministry of Micro, Small & Medium Enterprises
(MSME). It allows MSMEs to file their complaints online regarding delayed payments by
Central Ministries /Departments/ CPSEs/State Governments.
● The MSME SAMBANDH is the Public Procurement Portal launched by Central Government
for the MSME. It aims to monitor the implementation of the Public Procurement from MSEs
by Central Public Sector Enterprises.
● The MSME Sampark portal is a digital platform, wherein, jobseekers (passed out trainees /
students of 18 MSME Technology Centres and recruiters (various reputed national & multinational
companies) register themselves for getting employment and getting right kind of manpower
respectively.
Sources: https://samadhaan.msme.gov.in/MyMsme/MSEFC/MSEFC_Welcome.aspx

For More Study Material, Visit: studyiq.com Page no.


14
Schemes Test #2

https://www.financialexpress.com/industry/sme/msme-eodb-public-procurement-share-of-goods-
bought-from-msmes-hits-new-high-in-fy23-benefits-this-many-sellers/3019029/
http://sampark.msme.gov.in/?p=msmesampark

41. Answer: C
Explanation:
● Statement 1 is incorrect: India has introduced a draft to launch a global initiative 'Millet
International Initiative for Research and Awareness’(MIIRA) to encourage the consumption
and production of millet. It is in line with the UN declaring 2023 as the International Year of
Millets, the proposal for which was moved by India and supported by 72 countries.
● Statement 2 is incorrect: For MIIRA to take off, India will contribute the “seed money”, while
each G20 member will later have to contribute to its budget in the form of a membership fee. The
MIIRA secretariat will be in Delhi as India being the major producer of millets.
● Statement 3 is correct: MIIRA will aim to connect millet research organisations across the
world while also supporting research on these crops. Millets are small-grained cereals such as
sorghum (jowar), pearl millet (bajra), foxtail millet (kangni/ Italian millet), little millet (kutki), kodo
millet etc.These crops require much less water than rice and wheat and are mainly grown in rainfed
areas.
Source: https://indianexpress.com/article/explained/miira-global-initiative-g20-miillets-india-
importance-8469052/

42. Answer: A
Explanation:
● Statement (a) is correct: AKRUTI is an acronym for Advanced Knowledge
and Rural Technology Implementation initiative. Under this programme, several AKRUTI
MoUs were signed. Three AKRUTI nodes were set up in Maharashtra state and made operational
with funding by Government of Maharashtra and as a follow up, some more nodes were set up in
self-financed mode by other NGOs in other states. AKRUTI nodes through NGOs have
demonstrated the usefulness of BARC technologies for rural sector leading to societal benefit.
Further, it has demonstrated that technically oriented HR in rural sector is capable to deploy
technologies for their use under the guidance of BARC scientists and engineers. This
programme has potential to encourage village techno-preneurship based on BARC
technologies.
Source: India Year Book 2023.
https://www.barc.gov.in/akruti-tp/akrutitp.html

43. Answer: C
Explanation
● Statement 1 is correct: The central sector schemes are mainly formulated on subjects mainly from
the Union List. Central sector schemes are implemented by the central government and the
budget or financing needed for the scheme is entirely sourced by the central government.
o Centrally Sponsored Schemes – These are those schemes where there is financial participation
by both the center and states. The ratio of state participation may vary in 50:50, 60:40, 70:30,
75:25, or 90:10; showing higher contributions by the centre. Implementation of Centrally
Sponsored Scheme is made by State/UT Governments. Centrally Sponsored Schemes are
created on areas that are covered under the State List.
● Statement 2 is incorrect: As per the 2022 budget, there are 740 central sector schemes and
allocations. The Fifteenth Finance Commission in its report advised the centre to review and
rationalize the large number of central sector schemes to reduce unnecessary expenditure.
According to the Budget document, 130 centrally sponsored schemes spanning all ministries
have been “rationalised / revamped” into 65 schemes.
● Statement 3 is correct: Mahatma Gandhi National Rural Employment Generation Scheme is a
centrally sponsored scheme, while the National Rural Health Mission is a central sector scheme.
Sources:

For More Study Material, Visit: studyiq.com Page no.


15
Schemes Test #2

https://www.business-standard.com/about/what-is-centrally-sponsored-
schemes#:~:text=What%20is%20the%20difference%20between%20central%20sector%20schemes,ce
ntrally%20sponsored%20scheme%20is%20done%20by%20state%20governments.
https://www.indianeconomy.net/splclassroom/what-is-the-difference-between-central-sector-schemes-
and-centrally-sponsored-schemes/

44. Answer: B
Explanation:
● Statement 1 is correct: Ek Bharat Shreshtha Bharat under the Ministry of Tourism was
announced in 2015 on the occasion of the 140th birth anniversary of Sardar Vallabhbhai
Patel. It aims to celebrate cultural diversity through mutual interaction & reciprocity between
people of different States and UTs so that a spirit of understanding resonates throughout the
country. Every State and UT in the country is paired with another State/UT to carry out a structured
engagement with one another in the spheres of language, literature, cuisine, festivals, cultural
events, tourism etc.
● Statement 2 is incorrect: The Ministry of Tourism, as part of its ongoing promotional
activities, releases campaigns in the international and domestic markets under the ‘Incredible
India’ brand name, to promote various tourism destinations and products of India for
increasing foreign tourist arrivals and domestic travel within India.
● Statement 3 is correct: The Ministry of Tourism launched the ‘Dekho Apna Desh’ initiative
in January 2020 with the objective of creating awareness among the citizens about the rich
heritage and culture of the country and encouraging citizens to travel within the country,
Under this initiative, the Ministry organised a series of Webinars under the overall theme of Dekho
Apna Desh, showcasing the diverse culture, heritage, destinations and tourism products of the
country.
● Statement 4 is correct: Swadesh Darshan Scheme is a Central Sector scheme launched in
2014-15 by the Ministry of Tourism and Culture, Government of India for the integrated
development of theme-based tourist circuits of high tourist value, competitiveness and
sustainability in an integrated manner by synergising efforts to focus on needs and concerns of all
stakeholders to enrich tourist experience and enhance employment opportunities. Under it, 13
thematic circuits have been identified for development, namely: North-East India Circuit,
Buddhist Circuit, Himalayan Circuit, Coastal Circuit, Krishna Circuit, Desert Circuit, Tribal
Circuit, Eco Circuit, Wildlife Circuit, Rural Circuit, Spiritual Circuit, Ramayana Circuit and
Heritage Circuit.
Sources: India Year Book 2023.
https://pib.gov.in/PressReleasePage.aspx?PRID=1844726
https://tourism.gov.in/swadesh-darshan-scheme

45. Answer: B
Explanation:
● Pair 1 is correctly matched: The Ministry of Health & Family Welfare, Government of India,
under the National Health Mission launched the Rashtriya Bal Swasthya Karyakram
(RBSK). This program involves screening of children from birth to 18 years of age for 4 Ds-
Defects at birth, Diseases, Deficiencies and Development delays, spanning 32 common health
conditions for early detection and free treatment and management, including surgeries at
tertiary level.
● Pair 2 is incorrectly matched: The Ministry of Health and Family Welfare launched Rashtriya
Kishor Swasthya Karyakram (RKSK) on 7th January 2014 to reach out to 253 million
adolescents - male and female, rural and urban, married and unmarried, in and out-of-school
adolescents with special focus on marginalized and undeserved groups. It is health programme
for adolescents, in the age group of 10-19 years. The programme expands the scope of adolescent
health programming in India - from being limited to sexual and reproductive health, it now includes
in its ambit nutrition, injuries and violence (including gender based violence), non-communicable

For More Study Material, Visit: studyiq.com Page no.


16
Schemes Test #2

diseases, mental health and substance misuse. The strength of the program is its health promotion
approach.
● Pair 3 is incorrectly matched: The Pradhan Mantri Surakshit Matritva Abhiyan has been
launched by the Ministry of Health & Family Welfare (MoHFW), Government of India. The
program aims to provide assured, comprehensive and quality antenatal care, free of cost,
universally to all pregnant women on the 9th of every month.
● Pair 4 is correctly matched: "Mother’s Absolute Affection" is a nationwide programme of the
Ministry of Health and Family Welfare in an attempt to bring undiluted focus on promotion of
breastfeeding and provision of counselling services for supporting breastfeeding through health
systems. Under it early initiation and exclusive breastfeeding for first six months and
appropriate Infant and Young Child Feeding (IYCF) practices are promoted.
Sources:
https://pmsma.mohfw.gov.in/about-
scheme/#:~:text=What%20is%20the%20program%3F,the%209th%20of%20every%20month
https://vikaspedia.in/health/nrhm/national-health-programmes-1/rashtriya-kishor-swasthya-
karyakram-rksk
https://pib.gov.in/PressReleasePage.aspx?PRID=1809810

46. Answer: B
Explanation:
● Statement (b) is correct: A first-of-its kind database of arrested narcotics offenders has been made
operational for use. The portal, NIDAAN or the National Integrated Database on Arrested
Narco-offenders, has been developed by the Narcotics Control Bureau (NCB). It is part of the
narcotics coordination mechanism (NCORD) portal that was launched by Union Home Minister.
The NIDAAN platform sources its data from the ICJS (inter-operable criminal justice system) and
the e-Prisons (a cloud-based application) repository.
Source:
India Year Book 2023
https://www.india.gov.in/website-national-integrated-database-arrested-narco-offenders-nidaan

47. Answer: D
Explanation:
● Statement 1 is correct: PM VIKAS provides financial assistance to traditional artisans and
craftsmen. This support is not just monetary; it also encompasses skill development programs
designed to modernize and enhance the traditional skills these artisans possess. The objective is to
make these traditional crafts competitive and relevant, thereby increasing the income of the artisans.
● Statement 2 is correct: Eligibility of the scheme:
○ Any worker or artisan working in the unorganized sector on the basis of self-employment
with the help of work tools and their hands or working in 18 family-based traditional
businesses given in the scheme will be considered eligible for Vishwakarma Yojana.
○ Only a person who is 18 years of age or above 18 years can apply for the scheme.
○ Any person who wants to avail the benefits of this scheme should be engaged in the relevant
business on the date of registration and should not already be a beneficiary of schemes like
PMEGP, PM Swanidhi, and Mudra more than 5 years.
○ Only one member of a family can avail the benefit of PM Vishwakarma Yojana. This
includes husband, wife, and unmarried children living in the family.
○ A person employed in any government service (Central/State) and the family members of
that person cannot avail of the benefits of this scheme. This includes the husband and wife
and their unmarried children living in the family.
● Statement 3 is correct: Trades covered under Vishwakarma Yojana
○ Wood Based
■ Carpenter (Suthar)
■ Boat Maker
○ Iron/Metal Based/Stone Based

For More Study Material, Visit: studyiq.com Page no.


17
Schemes Test #2

■ Armourer
■ Blacksmith (Lohar)
■ Hammer and Tool Kit Maker
■ Locksmith
■ Sculptor (Moortikar, stone carver)
■ Stone Breaker
○ Gold/Silver Based
■ Goldsmith (Sunar)
○ Clay Based
■ Potter (Kumhaar)
○ Leather Based
■ Cobbler (Charmakar)
■ Shoesmith/Footwear Artisan
○ Architecture/Construction
■ Mason (Raajmistri)
○ Others
■ Doll & Toy Maker (Traditional)
■ Barber (Naai)
■ Garland Maker (Malakaar)
■ Washerman (Dhobi)
■ Tailor (Darzi)
■ Fishing Net Maker.

48. Answer: B
Explanation:
● Statement 1 is correct: Mahatma Gandhi NREGA is a centeally sponsored scheme, which
seeks to enhance the livelihood security of the households in rural areas of the country by providing
at least 100 days of guaranteed wage employment in every financial year to every household whose
adult members volunteer to do unskilled manual work.
● Statement 2 is incorrect: MGNREGA covers the entire country with the exception of districts
that have a hundred percent urban population.
● Statement 3 is correct: The implementation of the Act is done by the State Governments in
accordance with the Schemes formulated by them as per the provisions of the Act. All complaints
received in the Ministry are forwarded to the concerned States for taking appropriate action as per
law. In complaints of serious nature, the Ministry deputes National Level Monitors (NLMs) to
investigate the complaints. Reports of the NLMs are shared with the concerned State Governments
for taking corrective action.

49. Answer: D
Explanation:
● Statement 1 is correct: The National Water Mission is one of the 8 National Missions which
form the core of the National Action Plan for Climate Change. The objective of National Water
Mission is "conservation of water, minimizing wastage and ensuring its equitable distribution both
across and within States through integrated water resources development and management".
● Statement 2 is incorrect: The National Water Policy was formulated by Department of Water
Resources, River Development & Ganga Rejuvenation in 1987, which was subsequently
reviewed and revised in the year 2002 and 2012. The National Water Policy has been sent to all
States/ UTs for appropriate action. As per available information, 16 States/UT have formulated and
adopted their State Water Policies. With a goal to address the present challenges in water sector,
revision of National Water Policy 2012 has been envisaged by the Department of Water Resources,
River Development and Ganga Rejuvenation, Ministry of Jal Shakti and a drafting committee has
been constituted on 5th November, 2019 to revise the National Water Policy.
● Statement 3 is correct: Jal Shakti Abhiyan is a time-bound, mission-mode water conservation
campaign. The JSA will run in two Phases: Phase 1 from 1st July to 15th September 2019 for all

For More Study Material, Visit: studyiq.com Page no.


18
Schemes Test #2

States and Union Territories. And Phase 2 from 1st October to 30th November 2019 for States and
UTs receiving the retreating monsoon (Andhra Pradesh, Karnataka, Puducherry and Tamil Nadu).
Jal Shakti Abhiyan: Catch the Rain” (JSA:CTR) -2022, the third in the series of JSAs, has
been launched on 29.3.2022 to cover all the blocks of all districts (rural as well as urban areas)
across the country during 29th March 2022 to 30th November 2022 - the pre-monsoon and
monsoon period with a theme “where it falls, when it falls”.
● Statement 4 is correct: Central Ground Water Board is periodically monitoring the ground water
levels throughout India on a regional scale, through a network of monitoring wells. Central Ground
Water Board is periodically monitoring the ground water levels throughout India on a regional
scale, through a network of monitoring wells. Central Ground Water Authority (CGWA) has
been constituted under Section 3(3) of the ‘Environment (Protection) Act, 1986’ for the
purpose of regulation and control of ground water development and management in the Country.
Sources:
https://pib.gov.in/newsite/PrintRelease.aspx?relid=71513
https://pib.gov.in/PressReleasePage.aspx?PRID=1843395
https://pib.gov.in/PressReleasePage.aspx?PRID=1607166
https://pib.gov.in/PressReleasePage.aspx?PRID=1810544

50. Answer: C
Explanation:
● Statement 1 is correct: The National Clean Air Programme (NCAP) was launched by
the Ministry of Environment Forest & Climate Change (MoEF&CC) in 2019 as a long-term,
time-bound, national level strategy making determined effort to deal with the air pollution problem
across the country in a comprehensive manner.
● Statement 2 is incorrect: The aim envisaged to achieve targets of 20 to 30 %
reduction in Particulate Matter (PM) concentration by 2024 across the country. A new updated
target of 40% reduction in particulate matter concentration in cities covered under the (NCAP)
by 2026 has been set.
● Statement 3 is correct: NCAP covers 124 Non-Attainment Cities (NACSs) wherein National
Ambient Air Quality Standard (NAAQS) exceeded for 5 consecutive years, along with 8
other million plus cities of Fifteenth Finance Commission- Million Plus Cities Challenge Fund are
covered. Under NCAP, city-specific action plans have been prepared which include measures
for strengthening the air quality monitoring network, reducing vehicular and industrial
emissions, increasing public awareness, etc.
Source:
India Year Book 2023
https://www.studyiq.com/articles/air-pollution-and-ncap/

For More Study Material, Visit: studyiq.com Page no.


19
ECONOMY TEST #02

ECONOMY TEST #02


ANSWER KEY

Q. No. Answer Q. No. Answer Q. No. Answer Q. No. Answer Q. No. Answer
1 A 11 A 21 C 31 A 41 C
2 C 12 B 22 C 32 C 42 C
3 C 13 A 23 B 33 B 43 A
4 D 14 C 24 C 34 A 44 A
5 D 15 C 25 B 35 B 45 C
6 C 16 A 26 D 36 A 46 B
7 D 17 B 27 A 37 B 47 B
8 A 18 C 28 D 38 D 48 C
9 A 19 C 29 A 39 B 49 D
10 D 20 C 30 C 40 D 50 C

For More Study Material, Visit: studyiq.com Page no.


1
Schemes Test #2

Explanation

1. उत्तर: c
व्याख्या : ‘बााँध स्वास्थ्य एवं पु नवाास ननगरानी अनु प्रयोग’ (DHARMA) भारत के सभी बडे बांधों के संबंध
में बांध सुरक्षा संबंधी डे टा के प्रभावी संग्रह और प्रबंधन का समर्थ न करने के लिए एक वेब-आधाररत सॉफ्टवेयर
पैकेज है । सॉफ्टवेयर को केंद्रीय, राज्य और बां ध स्तर पर उपयोगकताथ ओं के लिए लडजाइन लकया गया है , लजसमें
उपयोगकताथ अनु मलत अलधकार उनके संबंलधत िाइसेंस द्वारा शालसत हैं ।
स्रोत : https://damsafety.cwc.gov.in/dharma/Home1/index.php

2. उत्तर: c
व्याख्या :
● तीनों सही हैं : गैल्वनाइलजं ग ऑगेलनक बायो-एग्रो ररसोसेज धन (GOBARdhan) भारत सरकार की एक
महत्वपूर्थ व्यापक पहि है , जो संपूर्थ सरकारी दृलिकोर् पर आधाररत है और इसका उद्दे श्य चक्रीय
अर्थ व्यवस्र्ा को बढावा दे ने की लदशा में अपलशि को धन में बदिना है ।
○ पे यजल एवं स्वच्छता नवभाग, जल शक्ति मंत्रालय गोबर धन (GOBARdhan) के नलए समन्वय
नवभाग है ।
○ गोबर धन (GOBARdhan) के उद्दे श्य :
■ गां वों को, उनके मवेलशयों और कृलि अपलशिों का सुरलित प्रबंधन और गां वों को स्वच्छ बनाने में
समर्थ न दे ना ।
■ उपचार प्रर्ालियों का उपयोग करके मवेलशयों और जै लवक अपलशि को धन/संपलि में पररवलतथत
करने में समु दायों का समर्थ न करना।
■ ग्रामीर् िे त्ों में उपयोग के लिए जै लवक अपलशि, लवशे िकर मवेलशयों के अपलशि को बायोगैस और
जै लवक खाद में पररवलतथत करना।
■ ग्रामीर् िे त्ों में अपलशि के प्रभावी लनपटान के माध्यम से पयाथ वरर्ीय स्वच्छता को बढावा दे ना और
वेक्टर (रोगवाहक) जलनत बीमाररयों पर अंकुश िगाना।
■ गोबर धन (GOBARdhan) इकाइयों की स्र्ापना, संचािन और प्रबंधन में उद्यलमयों, स्वयं सहायता
समू हों और युवा समू हों को शालमि करके ग्रामीर् रोजगार और आय सृजन के अवसरों को बढावा
दे ना।

3. उत्तर: d
व्याख्या :
अटि भू जि योजना (एबीएचवाई), लजसका नाम पूवथ प्रधान मं त्ी अटि लबहारी वाजपेयी के सम्मान में रखा गया है ,
एक योजना है , लजसका उद्दे श्य भू जि प्रबंधन में सुधार करना है । इसे जि जीवन लमशन के तहत 25 लदसंबर 2019
को िॉन्च लकया गया र्ा।
● कथन 1 सही नही ं है : अटल भूजल योजना अक्तिल भारतीय आधार पर लागू नही ं की गई है । इसके
बजाय, इसका ध्यान दे श के उन लवलशि िे त्ों पर केंलद्रत है , लजनकी पहचान गंभीर और अत्यलधक दोहन वािे
भू जि स्तर वािे िे त्ों के रूप में की गई है । यह योजना सात राज्यों- गुजरात, हररयार्ा, कनाथ टक, मध्य प्रदे श,
महारािर, राजस्र्ान और उिर प्रदे श में लकयाथ न्वित की जा रही है । इन राज्यों का चयन कई मानदं डों के आधार
पर लकया गया र्ा, लजसमें अत्यलधक दोलहत और गंभीर भू जि ब्लॉकों की व्यापकता और भू जि उपयोग का
स्तर शालमि र्ा।
● कथन 2 सही नही ं है : यह योजना पू री तरह से जल शक्ति मंत्रालय द्वारा कायााक्तन्वत की जाती है । अटि
भू जि योजना के कायाथ ियन के लिए पयाथ वरर्, वन और जिवायु पररवतथन मंत्ािय संयुक्त रूप से लजम्मेदार
नहीं है । यह योजना नवशेष रूप से भूजल की कमी की चुनौनतयों का समाधान करने और सतत भूजल
प्रबंधन प्रथाओं को बढावा दे ने के नलए बनाई गई है । इसमें जि प्रबंधन को सामू लहक लजम्मेदारी बनाने के
लिए सामु दालयक भागीदारी शालमि है । जि शन्वक्त मं त्ािय संस्र्ागत सुदृढीकरर्, िमता लनमाथ र् और सतत

For More Study Material, Visit: studyiq.com Page no.


1
Schemes Test #2

भू जि संसाधन प्रबंधन के लिए सामु दालयक स्तर पर व्यवहार पररवतथन को प्रोत्सालहत करने पर ध्यान केंलद्रत
करते हुए योजना के कायाथ ियन की दे खरे ख करता है ।
4. उत्तर: b
व्याख्या :
● कथन 1 सही नही ं है : बां ध पुनवाथ स और सुधार पररयोजना (केंद्रीय िे त् योजना) के चरर् 2 को सह-
लविपोलित लकया जा रहा है , िे लकन लवश्व बैंक और एलशयाई लवकास बैंक (एडीबी) दोनों लमिकर इसके
लविपोिर् में शालमि नहीं हैं । इसके स्र्ान पर, इसे लवश्व बैंक और एलशया अवसंरचना लनवेश बैंक
(एआईआईबी) द्वारा सह-लविपोलित लकया जाता है । इस सहयोग का उद्दे श्य भारत में बां ध सुरिा की महत्वपूर्थ
आवश्यकता को पूरा करने और जि संसाधन प्रबंधन में सुधार के लिए संसाधनों और लवशे िज्ञता को एकलत्त
करना है ।
● कथन 2 सही है : बां ध पुनवाथ स और सुधार पररयोजना (डीआरआईपी) का मु ख्य िक्ष्य भारत में इसमें शालमि
राज्यों में चयलनत मौजू दा बां धों की सुरिा और पररचािन प्रदशथ न में सुधार करना है । इसमें आवश्यक पुनवाथ स
उपायों को िागू करना, बां ध सुरिा लनगरानी और प्रबंधन प्रर्ाओं में सुधार करना और बां ध सुरिा के लिए
लजम्मेदार संस्र्ाओं के संस्र्ागत ढां चे और िमताओं को मजबूत करना शालमि है ।

5. उत्तर: c
व्याख्या :
● नवकल्प (c) है सही : ई-क्रां लत पहि लडलजटि इं लडया अलभयान का एक महत्वपूर्थ घटक है । इसका मु ख्य
उद्दे श्य कई तरीकों द्वारा एकीकृत और अंतर-संचालित प्रर्ालियों के माध्यम से नागररकों को इिे क्टरॉलनक
रूप से सरकारी सेवाएं प्रदान करके, सस्ती िागत पर ऐसी सेवाओं की दिता, पारदलशथ ता और लवश्वसनीयता
सुलनलित करके शासन को बदिने के लिए ई-गवनें स को पररवतथन करना है ।
● इस पहि में स्वास्थ्य, लशिा, कृलि जै से लवलभन्न िेत् शालमि हैं , लजसका उद्दे श्य आम आदमी के लिए सभी
सरकारी सेवाओं को सामान्य सेवा लवतरर् केंद्रों के माध्यम से उनके स्र्ानीय िे त् में सुिभ बनाना और
लकफायती िागत पर सेवाओं की दिता, पारदलशथ ता और लवश्वसनीयता सुलनलित करना है ।

6. उत्तर: d
व्याख्या :
● नवकल्प (d) सही है : भारत के रािरीय सुपरकंप्यूलटं ग लमशन (एनएसएम) का िक्ष्य 70 से अलधक उच्च-प्रदशथ न
कंप्यूलटं ग सुलवधाओं से युक्त एक लवशाि सुपरकंप्यूलटं ग लग्रड स्र्ालपत करके दे श भर में फैिे रािरीय शै िलर्क
और अनु संधान एवं लवकास संस्र्ानों को सशक्त बनाना है। इन सुपर कंप्यूटरों का उद्दे श्य लवलभन्न िे त्ों में
अनु संधान और लवकास को सुलवधाजनक बनाना है , लजससे जलटि समस्याओं को हि करने में भारत की
िमताओं में उल्ले खनीय वृन्वि होगी। रािरीय सुपरकंप्यूलटं ग लमशन (एनएसएम) के अनु प्रयोग िे त्ों में जलवायु
मॉडनलंग (जलवायु प्रनतरूपण), मौसम की भनवष्यवाणी, एयरोस्पे स इं जीननयररं ग (अं तररक्ष
अनभयांनत्रकी), कम्प्यूटेशनल बायोलॉजी (अनभकलनात्मक जीव नवज्ञान), आणनवक गनतशीलता और
नवत्त शालमि हैं ।

7. उत्तर: b
व्याख्या :
प्रधानमं त्ी ग्रामीर् लडलजटि सािरता अलभयान (पीएमजीलदशा) भारत सरकार द्वारा शु रू की गई एक योजना है
लजसका उद्दे श्य दे श के ग्रामीर् नागररकों को लडलजटि सािरता प्रदान करना है ।
● कथन 1 सही नही ं है : पीएमजीलदशा योजना 14 से 60 विथ की आयु की ग्रामीर् आबादी को पूरा करने के
लिए बनाई गई है ।
○ इसमें स्कूिी बच्चे, कॉिेज के छात्, हालशए पर रहने वािे समु दाय, लकसान, कारीगर और सामान्य
रूप से ग्रामीर् कायथबि शालमि हैं ।
● कथन 2 सही है : यह कायथक्रम वास्तव में ग्रामीर् आबादी के लवलभन्न समू हों को प्रार्लमकता दे ता है , लजसमें
गैर-स्माटथ फोन उपयोगकताथ भी शालमि हैं , लजनके पास कंप्यूटर या इं टरने ट तक सीलमत या कोई पहुं च नहीं
है ।

For More Study Material, Visit: studyiq.com Page no.


2
Schemes Test #2

○ इस पहि का उद्दे श्य लडलजटि लवभाजन को पाटना और लडलजटि समावेशन को बढावा दे ना है ।


○ गैर -स्माटथ फोन उपयोगकताथ ओं पर ध्यान केंलद्रत करके, यह कायथक्रम उनके कौशि को उन्नत करने
और उन्हें संचार के लिए लडलजटि उपकरर्ों का उपयोग करने , सरकारी सेवाओं की एक श्ृं खिा
तक पहुं चने , नकदरलहत िे नदे न करने आलद में सिम बनाने का प्रयास करता है ।

8. उत्तर: b
व्याख्या :
● कथन 1 और 2 सही हैं : सॉफ्टवेयर टे क्नोिॉजी पाकथ (एसटीपी) योजना कंप्यूटर सॉफ्टवेयर के लवकास और
लनयाथ त के लिए 100% लनयाथ त-उन्मुख योजना है , लजसमें संचार लिं क या भौलतक मीलडया का उपयोग करके
पेशेवर सेवाओं का लनयाथ त भी शालमि है । एक अनू ठी योजना के रूप में , यह एक िे त् अर्ाथ त कंप्यूटर
सॉफ्टवेयर पर केंलद्रत है ।
● कथन 3 सही नही ं है : 100% लवदे शी इन्विटी की अनु मलत है ।
● कथन 4 सही है : सॉफ्टवेयर टे क्नोिॉजी पाकथ (एसटीपी) इकाइयों में हाडथ वेयर और सॉफ्टवेयर के सभी
आयात पूरी तरह से शु ल्क मुक्त हैं , पुरानी पूंजीगत वस्तु ओं के आयात की भी अनु मलत है ।

9. उत्तर: d
व्याख्या :
● नवकल्प (d) सही है : 'मे घराज' (MeghRaj) भारत सरकार की पहि है , लजसे नागररकों को सरकारी सेवाओं
के कुशि लवतरर् के लिए क्लाउड कंप्यूलटं ग तकनीक का िाभ उठाने के लिए शु रू लकया गया है । 'मे घराज'
(MeghRaj) का उद्दे श्य बहुआयामी है , जो मु ख्य रूप से सूचना और संचार प्रौद्योलगकी (आईसीटी) बुलनयादी
अवसंरचना का इितम उपयोग सुलनलित करते हुए और िागत को कम करते हुए ई-गवनें स सेवाओं की
दिता, िचीिापन और मापनीयता को बढाने पर ध्यान केंलद्रत करता है ।
● रािरीय सूचना लवज्ञान केंद्र (एनआईसी) मे घराज (MeghRaj) पहि के तहत रािरीय क्लाउड सेवाएं प्रदान कर
रहा है । प्रदान की जाने वािी सेवाएं इस प्रकार हैं ।
○ एक सेवा के रूप में अवसंरचना (IaaS) : IaaS आपको ब्लैं क वीएम से जु डे सीपीयू, मेमोरी, लडस्क
स्टोरे ज जै से बुलनयादी वचुथअि कंप्यूट इं फ्रास्टर क्चर संसाधन प्रदान करता है , जो आपको स्क्रैच और
कस्टमाइजेशन से आईएसओ का उपयोग करके ओएस स्र्ालपत करने की अनु मलत दे ता है । हािााँ लक
आपको ओएस और एन्विकेशन सॉफ़्टवेयर (यलद कोई हो) के लिए अपने स्वयं के िाइसेंस का
उपयोग करना होगा।
● एक सेवा के रूप में प्लेटफॉमा (PaaS) : PaaS पूवथ-स्र्ालपत वेब और डे टाबेस सवथर प्रदान करता है , तालक
आप सवथर सेटअप के बारे में लचंता लकए लबना वेब एन्विकेशन प्रकालशत और चिा सकें। सवथर बुलनयादी सुरिा
व्यवस्र्ा के सार् पहिे से तैयार कॉन्वफ़िगर लकए गए हैं । सवथरों को त्वररत रूप से तैनात करने और अपने वेब
एन्विकेशन प्रकालशत करने के लिए PaaS सेवा का उपयोग लकया जा सकता है । ओएस और एन्विकेशन
सॉफ़्टवेयर िाइसेंस हमारे द्वारा प्रस्ताव के भाग के रूप में प्रदान लकए जाते हैं ।
● एक सेवा के रूप में सॉफ़्टवे यर (SaaS) : यह ऑन लडमां ड सॉफ़्टवेयर सेवा प्रदान करता है । SaaS एक
सॉफ्टवेयर लवतरर् प्रलतमान है , जहां उपयोगकताथ एन्विकेशन या लकसी भी घटक का समर्थ न करने के लिए
लजम्मेदार नहीं होता हैं । सवथर इं फ्रास्टर क्चर, ओएस और सॉफ्टवेयर का प्रबंधन क्लाउड सेवाओं द्वारा लकया जा
रहा है । यलद आपके पास वेब एन्विकेशन है और आप इसे उपयोगकताथ ओं को लवतररत करना चाहते हैं , तो
सॉफ़्टवेयर के माध्यम से सेवा के रूप में लवतररत करने के लिए हमारी क्लाउड सेवा का उपयोग लकया जा
सकता है ।
● एक सेवा के रूप में भंडारण (STaaS) : STaaS आवश्यकता आधाररत भं डारर् समाधान प्रदान करता है ।
यह पारं पररक ऑन-साइट और समलपथत भंडारर् प्रर्ालियों के लिए उत्कृि लवकल्प प्रदान करता है और कई
भं डारर् स्तरों को तैनात करने और प्रबंलधत करने की जलटिताओं को कम करता है । आप इसका उपयोग
आपदा िलतपूलतथ में जोन्वखमों को कम करने , ररकॉडथ के लिए दीघथकालिक प्रलतधारर् प्रदान करने और लनरं तरता
और उपिब्धता दोनों को बढाने के लिए कर सकते हैं ।
● होक्तटंग वातावरण : एनआईसी क्लाउड सलवथसेज वचुथअि मशीन बनाने के लिए 3 अिग-अिग प्रकार के
वातावरर् प्रदान करती है अर्ाथ त उत्पादन, स्टे लजं ग और लवकास, तालक आप अपने वीएम को अिग रखें और

For More Study Material, Visit: studyiq.com Page no.


3
Schemes Test #2

PaaS और IaaS सेवा मॉडि दोनों के लिए व्यावसालयक आवश्यकता के आधार पर उन्हें ठीक से प्रबंलधत
करें ।

10. उत्तर: c
व्याख्या : लडलजटि इं लडया के नौ स्तं भों में शालमि हैं :
● ब्रॉडबैं ड राजमागा
● मोबाइल कने क्तिनवटी तक सावा भौनमक पहं च
● सावथजलनक इं टरने ट पंहुच कायथक्रम कायथक्रम
● ई-गवनें स : प्रौद्योलगकी के माध्यम से सरकार में सुधार
● ई-क्रां लत : सेवाओं का इिेक्टरॉलनक लवतरर्
● सभी के लिए सूचना
● इलेिरॉननक्स नवननमााण
● नौकररयों के नलए सूचना प्रौद्योनगकी
● प्रारं लभक फसि कायथक्रम
स्रोत : https://digitalindia.gov.in/programme-pillars/

11. उत्तर: b
व्याख्या:
● कथन 1 सही नही ं है : लसक्योर लहमािय पररयोजना "सतत लवकास के लिए वन्यजीव संरिर् और अपराध
रोकर्ाम पर वैलश्वक साझेदारी" (वैलश्वक वन्यजीव कायथक्रम) का एक लहस्सा है ।
o इसे 2017 में पयाथ वरर्, वन और जिवायु पररवतथन मंत्ािय (एमओईएफसीसी) और संयुक्त रािर लवकास
कायथक्रम (यूएनडीपी) के बीच एक संयुक्त पररयोजना के रूप में िॉन्च लकया गया र्ा और वैलश्वक
पयाथ वरर् सुलवधा (जीईएफ) द्वारा लवि पोलित लकया गया र्ा।
● कथन 2 सही है : उद्दे श्य:
o दीघथकालिक पाररन्वस्र्लतकी तंत् िचीिापन, आवास कने न्वक्टलवटी और लहम तेंदुए एवं अन्य िु प्तप्राय
प्रजालतयों और उनके आवासों के संरिर् को सुरलित करने के लिए प्रमु ख जै व लवलवधता िे त्ों का संरिर्
और उनका प्रभावी प्रबंधन।
o उच्च श्े र्ी के लहमाियी पाररन्वस्र्लतकी तंत् में सतत सामु दालयक आजीलवका और प्राकृलतक संसाधन
प्रबंधन को सुरलित करना।
o वन्यजीव अपराध और संबंलधत खतरों को कम करने के लिए प्रवतथन, लनगरानी और सहयोग बढाना।
o ज्ञान, वकाित, संचार और सूचना प्रर्ालियााँ स्र्ालपत करना।

12. उत्तर: b
व्याख्या:
● जिवायु पररवतथन पर रािरीय कायथयोजना (NAPCC), 30 जू न 2008 को प्रारं भ की गई र्ी।
● यह एक रािरीय रर्नीलत की रूपरे खा तैयार करता है , लजसका उद्दे श्य दे श को जिवायु पररवतथन के अनु कूि
होने में सिम बनाना तर्ा भारत के लवकास पर् की पाररन्वस्र्लतक संधारर्ीयता में वृन्वि करना है ।
● आठ रािरीय लमशन, जो जिवायु पररवतथन पर रािरीय कायथयोजना (NAPCC) का लहस्सा हैं , वे हैं :
1. रािरीय सौर लमशन
2. उन्नत ऊजाथ दिता के लिए रािरीय लमशन
3. सतत आवास पर रािरीय लमशन
4. रािरीय जि लमशन
5. ने शनि लमशन ऑन सस्टे लनं ग लहमाियन ईकोलसस्टम
6. हररत भारत के लिए रािरीय लमशन
7. सतत कृलि के लिए रािरीय लमशन
8. जिवायु पररवतथन हे तु रर्नीलतक ज्ञान पर रािरीय लमशन

For More Study Material, Visit: studyiq.com Page no.


4
Schemes Test #2

13. उत्तर: b
व्याख्या:
 कथन 1 सही नही ं है : उद्योग संक्रमर् के लिये ने तृत्व समू ह (LeadIT) उन दे शों और कंपलनयों को एकत्
करता है , जो पेररस समझौते को प्राप्त करने के लिए कारथ वाई हे तु प्रलतबि हैं ।
o इसे लसतंबर 2019 में , संयुक्त रािर जिवायु कारथ वाई लशखर सम्मेिन में स्वीडन एवं भारत की सरकारों
द्वारा प्रारं भ लकया गया र्ा।
 कथन 2 और 3 सही हैं : LeadIT (उद्योग संक्रमर् के लिये ने तृत्व समू ह) पहि कलठन औद्योलगक िे त् के लनम्न
काबथन संक्रमर् पर केंलद्रत है तर्ा लवश्व आलर्थ क मं च द्वारा समलर्थ त है ।

14. उत्तर: a
व्याख्या:
 कथन 1 और 3 सही नही ं हैं : प्रधानमं त्ी सूक्ष्म खाद्य प्रसंस्करर् उद्यम (PM-FME) योजना, एक केंद्रीय
प्रायोलजत योजना है , लजसका उद्दे श्य सूक्ष्म-उद्यमों के समि आने वािी चुनौलतयों का समाधान करना तर्ा इन
उद्यमों के उन्नयन एवं औपचाररकीकरर् के समर्थ न में समू हों और सहकारी सलमलतयों की िमता का दोहन
करना है । िगभग 2 िाख सूक्ष्म खाद्य प्रसंस्करर् उद्यमों को, क्रेनडट नलंक्ड सक्तिडी के रूप में प्रत्यि
सहायता प्रदान की जाएगी।
o इस योजना में 2020-21 से 2024-25 तक, पां च विों की अवलध में 10,000 करोड रुपये के पररव्यय की
पररकल्पना की गई है । इस योजना के अंतगथत होने वािा व्यय, केंद्र और राज्य सरकारों के बीच 60:40
अनु पात में तर्ा उिर- पूवी और लहमाियी राज्यों में 90:10 अनु पात में , लवधालयका वािे केंद्र शालसत
प्रदे शों के सार् 60:40 के अनु पात में तर्ा अन्य केंद्र शालसत प्रदे शों के लिए केंद्र द्वारा 100% लदया जाएगा।
 कथन 2 सही है : प्रधानमं त्ी सूक्ष्म खाद्य प्रसंस्करर् उद्यम (PM-FME) योजना की लवशे िताएं इस प्रकार हैं :
o एक नजला एक उत्पाद: यह योजना आदानों की खरीद, सामान्य सेवाओं का िाभ उठाने एवं उत्पादों के
लवपर्न के संबंध में स्केि का िाभ प्राप्त करने के लिए, एक लजिा एक उत्पाद (ODOP) के घटक को
अपनाती है ।
▪ राज्य सरकार मौजूदा समूहों और कच्चे माल की उपलब्धता को ध्यान में रिते हए, एक
नजले के नलए एक उत्पाद की पहचान करे गी।
o प्राथनमकता वाले क्षे त्र: यह योजना अपलशि से संपदा उत्पाद, िघु वन उत्पाद और आकां िी लजिों पर
केंलद्रत है ।
o नोडल बैं क- यूलनयन बैंक ऑफ इं लडया।
स्रोत:
https://pib.gov.in/PressReleasePage.aspx?PRID=1984009

15. उत्तर: a
व्याख्या:
 कथन 1 सही है : प्रधानमं त्ी मत्स्य संपदा योजना (PMMSY) दो लभन्न-लभन्न घटकों वािी एक व्यापक योजना
है , अर्ाथ त्- (a) केंद्रीय िे त् योजना (CS) और (b) केंद्र प्रायोलजत योजना (CSS)। केंद्र प्रायोलजत योजना (CSS)
घटक को लनम्नलिन्वखत तीन व्यापक शीिथ कों के अंतगथत गैर-िाभार्ी उन्मुख और िाभार्ी उन्मु ख उप-
घटकों/गलतलवलधयों में लवभालजत लकया गया है :
(i) उत्पादन और उत्पादकता में वृन्वि (ii) बुलनयादी अवसंरचना और फसि कटाई के बाद प्रबंधन (iii) ) मत्स्य
पािन प्रबंधन तर्ा लनयामक अवसंरचना
 कथन 2 सही नही ं है : यह योजना सम्पू र्थ दे श के सभी राज्यों/केंद्रशानसत प्रदे शों (तटीय एवं भू -आबि
राज्यों) में िागू की जाएगी।
 कथन 3 सही है : प्रधानमं त्ी मत्स्य संपदा योजना (PMMSY) के उद्दे श्य-
1. मत्स्य पािन िेत् की िमता का सतत, लजम्मेदार, समावेशी एवं न्यायसंगत तरीके से दोहन करना।
2. भू लम और जि के लवस्तार, सघनीकरर्, लवलवधीकरर् तर्ा उत्पादक उपयोग के माध्यम से, मत्स्य उत्पादन
एवं उत्पादकता में वृन्वि करना।
3. फसि कटाई के बाद प्रबंधन और गुर्विा सुधार सलहत, मू ल्य श्ृंखिा को आधुलनक तर्ा मजबूत करना।

For More Study Material, Visit: studyiq.com Page no.


5
Schemes Test #2

4. मछु आरों और मछिी पािकों की आय दोगुनी करना और सार्थ क रोजगार उत्पन्न करना।
5. कृलि सकि मूल्यवधथन (GVA) और लनयाथ त में मत्स्य पािन िे त् के योगदान को बढाना।
6. मछु आरों और मछिी पािकों के लिए सामालजक, भौलतक एवं आलर्थ क सुरिा सुलनलित करना।
7. एक मजबूत मत्स्य प्रबंधन एवं लनयामक ढां चे का लनमाथ र् करना।
स्रोत: https://static.pib.gov.in/WriteReadData/userfiles/PMMSY%20BookEnglish.pdf

16. उत्तर: a
व्याख्या:
 कथन 1 सही है : MSME (सूक्ष्म, िघु तर्ा मध्यम उद्यम) के प्रदशथ न को बेहतर और तेज करना (RAMP),
एक केंद्रीय क्षे त्र की योजना है तर्ा इसका िक्ष्य मौजू दा MSME योजनाओं के प्रभाव को बढाने के सार्,
राज्यों में MSME की कायाथ ियन िमता तर्ा कवरे ज को बढाना है ।
 कथन 2 सही है : इस योजना के नलए नवश्व बैं क द्वारा सहायता प्रदान की जा रही है ।
 योजना के घटक:
o भुगतान से जुडे संकेतक (DLI)-लविं लबत भु गतान की घटनाओं को कम करने , MSME त्वरर् में केंद्र-
राज्यों के मध्य बेहतर सहयोग, MSME योजनाओं की प्रभावशीिता को बढाने आलद जै से कई संकेतकों
को पूरा करने पर, लवश्व बैंक की ओर से लनष्किथ प्रदान लकए जाएं गे।
o रणनीनतक ननवेश योजनाएाँ (SIP) - RAMP योजना का एक महत्वपूर्थ घटक, SIP की तैयारी है । इसमें
MSME को संगलठत करने , प्रमु ख बाधाओं की पहचान करने , प्रार्लमकता िे त् के हस्तिे प के लिए मानक
लनधाथ ररत करने आलद की योजना को शालमि लकया गया है ।
● कथन 3 सही नही ं है : RAMP योजना हे तु पात् होने के लिए, MSME को सूक्ष्म, िघु तर्ा मध्यम उद्यम लवकास
(MSMED) अलधलनयम, 2006 के अंतगथत पंजीकृत होना चालहए तर्ा उसके पास एक वैध उद्योग आधार संख्या
(UAN) होना चालहए। MSME का टर ै क ररकॉडथ और क्रेलडट रे लटं ग भी अच्छी होनी चालहए।
स्रोत:
https://pib.gov.in/PressReleasePage.aspx?PRID=1811360

17. उत्तर: c
व्याख्या:
 कथन 1 सही है और कथन 2 सही नही ं है : राष्ट्रीय हररत हाइडरोजन नमशन का उद्दे श्य, 2047 तक भारत
को ऊजाथ के िे त् में आत्मलनभथर बनाना तर्ा 2070 तक शु ि शू न्य िक्ष्य प्राप्त करना है ।
o अन्य उद्दे श्य:
■ हररत हाइडरोजन के व्यावसालयक उत्पादन को प्रोत्साहन प्रदान करना और भारत को ईंधन का शुि
लनयाथ तक बनाना
■ ऊजाथ िे त् को काबथन मुक्त करना तर्ा आयालतत जीवाश्म ईंधन पर लनभथ रता कम करना
■ हररत हाइडरोजन की मां ग सृजन, उत्पादन और उपयोग को सुलवधाजनक बनाना
■ हररत हाइडर ोजन और उसके डे ररवे नटव के नलए ननयाात के अवसर उत्पन्न करना
■ स्वदे शी लवलनमाथ र् िमताओं का लवकास करना
 कथन 3 सही है : ऑयि इं लडया लिलमटे ड (OIL) ने असम में अपने जोरहाट पंप स्टे शन पर प्रलत लदन 10
लकिोग्राम की स्र्ालपत िमता के सार्, भारत के पहिे 99.999% शु ि हररत हाइडरोजन पायिट िां ट की
शु रुआत के सार् भारत में हररत हाइडरोजन अर्थव्यवस्र्ा की लदशा में पहिा महत्वपूर्थ कदम उठाया है । यह
संयंत् 3 माह के ररकॉडथ समय में स्र्ालपत लकया गया है ।
स्रोत:
https://pib.gov.in/PressReleasePage.aspx?PRID=1888547

18. उत्तर: a
व्याख्या:
 कथन 1 सही है : सौर शहर नवकास योजना, सौर शहरों के लवकास के लिए नगर लनगमों को सहायता
प्रदान करके शहरी िे त्ों में नवीकरर्ीय ऊजाथ के उपयोग को बढावा दे ने की एक पहि है । इसका िक्ष्य प्रलत

For More Study Material, Visit: studyiq.com Page no.


6
Schemes Test #2

शहर/कस्बे को ₹ 50 िाख तक की लविीय सहायता प्रदान करना है ।


शहरों की पहचान के नलए मानदं ड-
o 50,000 से िे कर 50 िाख तक की जनसंख्या वािा शहर
o ऊजाथ दिता और नवीकरर्ीय ऊजाथ को बढावा दे ने वािा शहर
 कथन 2 सही नही ं है : नवीन और नवीकरणीय ऊजाा मंत्रालय, इस योजना को कायााक्तन्वत करने के
नलए नोडल मंत्रालय है । कुि 60 शहरों को सौर शहरों के रूप में लवकलसत करने हेतु सहायता दे ने की
स्वीकृलत प्रदान की गई है ।
 कथन 3 सही नही ं है : मध्य प्रदे श का सााँची, भारत का पहला सौर शहर है ।
o गु जरात का मोढे रा, दे श का पहला सौर गांव है ।
स्रोत:
https://pib.gov.in/Pressreleaseshare.aspx?PRID=1519372
https://newsonair.gov.in/Main-News-Details.aspx?id=467334

19. उत्तर: c
व्याख्या:
 कथन 1 सही नही ं है , जबनक कथन 2, 3 और 4 सही हैं : भारत सरकार भारतीय िोगों में एनीलमया को
कम करने के िक्ष्य के सार् पोिर् अलभयान के अंतगथत, एनीलमया मुक्त भारत (AMB) रर्नीलत को िागू
करती है । एनीलमया मु क्त भारत रर्नीलत को विथ 2018 और 2022 के मध्य बच्चों, लकशोरों तर्ा प्रजनन आयु
वगथ (15-49 विथ ) में मलहिाओं के बीच प्रलत विथ एनीलमया की व्यापकता को 3% कम करने के लिए संरलचत
लकया गया है । सरकार का उद्दे श्य 6X6X6 रर्नीलत के माध्यम से लनवारक और उपचारात्मक तंत् प्रदान करना
है , लजसमें रर्नीलत को िागू करने के लिए सभी लहतधारकों हे तु छह िलित िाभार्ी, छह हस्तिे प तर्ा छह
संस्र्ागत तंत् शालमि हैं ।
एनीलमया मुक्त भारत (AMB) रर्नीलत के अंतगथत सरकार द्वारा लकये गये प्रमु ख हस्तिे पों में शालमि हैं :
o रोगलनरोधी आयरन तर्ा फोलिक एलसड पूरकता।
o गहन विथ -भर व्यवहार पररवतथन संचार (BCC) अलभयान तर्ा लविं लबत कॉडथ क्लैन्वम्पंग
o बच्चों और लकशोरों की लनधाथ ररत अवलधयों पर कृलम मु न्वक्त।
o लडलजटि तरीकों और दे खभाि उपचार लबंदु का उपयोग करके, एनीलमया का परीिर्।
o मिे ररया, हीमोग्लोलबनोपैर्ी और फ्लु ओरोलसस पर लवशेि ध्यान दे ने के सार्, स्र्ालनक बन्वस्तयों में एनीलमया
के गैर-पोिर् संबंधी कारर्ों का समाधान करना।
o संबंलधत लवभागों तर्ा अन्य मं त्ाियों के सार् अलभसरर् एवं समिय।
o स्वास्थ्य दे खभाि प्रदाताओं की िमता लनमाथ र् के लिए एनीलमया लनयंत्र् पर रािरीय उत्कृिता केंद्र और
उन्नत शोध को शालमि करना।
o एनीलमया मुक्त भारत डै शबोडथ का उपयोग करके, राज्यों/केंद्र शालसत प्रदे शों में प्रगलत की लनगरानी
करना।
स्रोत: संघ िोक सेवा आयोग, लसलवि सेवा प्रारं लभक परीिा- 2023

20. उत्तर: b
व्याख्या:
 कथन 1 सही नही ं है : जननी सुरिा योजना (JSY), रािरीय ग्रामीर् स्वास्थ्य लमशन (NHM) के अंतगथत एक
सुरलित मातृत्व योजना है । रािरीय मातृत्व िाभ योजना (NMBS) को संशोलधत करके अप्रैि 2005 में , जननी
सुरिा योजना प्रारं भ की गई र्ी।
 कथन 2 और 3 सही हैं : गभथ वती मलहिाओं में स्वास्थ्य केन्द्ों में प्रसव को बढावा दे कर मातृ एवं लशशु मृ त्यु
दर को कम करने के उद्दे श्य से, जननी सुरिा योजना (JSY) कायाथ न्वित की जा रही है । यह योजना सभी राज्यों
तर्ा केंद्र शालसत प्रदे शों (UTs) में कायाथ ियन के अधीन है , लजसमें कम प्रदशथ न करने वािे राज्यों पर लवशे ि
ध्यान लदया गया है । जननी सुरिा योजना (JSY) एक केंद्र प्रायोलजत योजना है , जो लक प्रसव तर्ा प्रसव के बाद
की दे खभाि के सार् नकद सहायता को एकीकृत करती है । इस योजना ने सरकार और गभथ वती मलहिाओं
के मध्य एक प्रभावी कडी के रूप में , मान्यता प्राप्त सामालजक स्वास्थ्य कायथकताथ (आशा) की पहचान की है ।

For More Study Material, Visit: studyiq.com Page no.


7
Schemes Test #2

 कथन 4 सही नही ं है : यह योजना उन राज्यों के लिए, जै से लक- उिर प्रदे श, उिराखंड, लबहार, झारखं ड,
मध्य प्रदे श, छिीसगढ, असम, राजस्र्ान, उडीसा और जम्मू तर्ा कश्मीर राज्य, जहां स्वास्थ्य केन्द्ों में प्रसव
दर कम है , लवशेि छूट के सार् लनधथन गभथ वती मलहिाओं पर ध्यान केंलद्रत करती है । यह योजना गभथवती
मलहिाओं के मध्य स्वास्थ्य केन्द्ों में प्रसव को बढावा दे ने के लिए, आशा (मान्यताप्राप्त सामालजक स्वास्थ्य
कायथकताथ ) के नाम से जानी जाने वािी मलहिा स्वास्थ्य स्वयंसेवकों को प्रदशथ न-आधाररत प्रोत्साहन भी प्रदान
करती है । इस पहि के अंतगथत, पात् गभथ वती मलहिाएं सीधे अपने बैंक खातों में जननी सुरिा योजना (JSY)
िाभ प्राप्त करने की अलधकारी होती हैं ।
स्रोत: संघ िोक सेवा आयोग, लसलवि सेवा प्रारं लभक परीिा- 2023

21. उत्तर: d
व्याख्या:
कथन 1 सही है : सां सद स्र्ानीय िे त् लवकास योजना (एमपीएिएडीएस) एक चािू योजना है । सां सद स्र्ानीय िेत्
लवकास योजना एक केंद्रीय िे त् की योजना है , जो पूरी तरह से भारत सरकार द्वारा लवि पोलित है और इसे महामारी
के दौरान कुछ समय के लिए लनिं लबत कर लदया गया र्ा। यह योजना सांसदों को अपने ननवााचन क्षे त्रों में
पे यजल, नशक्षा, सावा जननक स्वास्थ्य, स्वच्छता, सडक आनद जैसी स्थायी सामुदानयक संपनत्तयों के ननमााण
के नलए कायों की नसफाररश करने में सक्षम बनाती है । व्यक्तिगत लाभ और चल एवं अस्थायी संपनत्तयां
(कुछ को छोडकर) आम तौर पर इस योजना के अं तगा त सक्तिनलत नही ं है ।
कथन 2 सही है : यह योजना अनु सूलचत जालत और अनु सूलचत जनजालत आबादी वािे िे त्ों के लवकास पर लवशे ि
ध्यान दे ती है । सांसदों को अनु सूनचत जानत आबादी वाले क्षे त्रों के नलए वानषाक सांसद स्थानीय क्षे त्र नवकास
योजना पात्रता का कम से कम 15% और अनु सूनचत जनजानत आबादी वाले क्षे त्रों के नलए 7.5% लागत
वाले कायों की नसफाररश करनी होती है ।
कथन 3 सही नही ं है : प्रत्येक सां सद को अपने लनवाथ चन िे त्ों में सामु दालयक उपयोग के लिए पररयोजनाओं पर
वालिथ क रूप से ₹5 करोड का खचथ करने का अलधकार होता है । िचा न की गई रानश को ननवााचन क्षे त्र में नजला
प्रानधकारी के साथ एक गै र-व्यपगत िाते में रिा जाता है और बाद के वषों में इसका उपयोग नकया जा
सकता है ।
कथन 4 सही है : संसद सदस्य स्र्ानीय िे त् लवकास योजना के तहत सां सदों को सीधे फंड नहीं लमिता है । लकसी
अनु शंलसत पररयोजना को मंजूरी लमिने के बाद केंद्र सीधे स्वीकृत रालश को 2.5 करोड रुपये की दो लकस्तों में
संबंलधत एमपी के नोडि लजिे के लजिा अलधकाररयों को हस्तां तररत करता है । नजला प्रानधकरण नजला स्तर पर
योजना के तहत कायों के समग्र समन्वय और पयावेक्षण के नलए नजिे दार होगा और प्रत्येक वषा कायाान्वयन
के तहत कम से कम 10% कायों का ननरीक्षण करे गा। नजला प्रानधकरण को यथासं भव पररयोजनाओं के
ननरीक्षण में सांसदों को शानमल करना चानहए।
स्रोत: संघ िोक सेवा आयोग, लसलवि सेवा प्रारं लभक परीिा- 2020

22. उत्तर: c
व्याख्या:
कथन 1 सही नही ं है : प्रधानमंत्री कौशल नवकास योजना (पीएमकेवीवाई) राष्ट्रीय कौशल नवकास ननगम
द्वारा कायााक्तन्वत कौशल नवकास और उद्यनमता मंत्रालय (एमएसडीई) की प्रमुि योजना है ।
कथन 2 सही है : इसका उद्दे श्य युवाओं को अल्पकालिक कौशल नवकास प्रनशक्षण, सॉफ्ट कौशि, उद्यलमता,
लविीय, लडलजटि सािरता और प्रमार्न में प्रलशिर् प्रदान करना और उन्हें दे श भर में बेहतर आजीलवका के लिए
रोजगार योग्य बनाना है ।
कथन 3 सही है : इस कौशि प्रमार्न योजना का उद्दे श्य बडी संख्या में भारतीय युवाओं को उद्योग-प्रासंलगक
कौशि प्रलशिर् िे ने में सिम बनाना है , जो उन्हें बेहतर आजीलवका प्राप्त करने में सहायता करे गा। इसका उद्दे श्य
दे श के अलवलनयलमत कायथबि की दिताओं को रािरीय कौशि योग्यता ढां चे (एनएसक्यूएफ) के अनु रूप बनाना
है ।
स्रोत: संघ िोक सेवा आयोग, लसलवि सेवा प्रारं लभक परीिा- 2018

For More Study Material, Visit: studyiq.com Page no.


8
Schemes Test #2

23. उत्तर: c
व्याख्या:
नवकल्प (c) सही है : एकीकृत जिसम्भर लवकास कायथक्रम ग्रामीण नवकास मंत्रालय के भूनम संसाधन नवभाग
द्वारा कायााक्तन्वत नकया जाता है ।
इस कायाक्रम के मुख्य उद्दे श्य इस प्रकार हैं :
● लमट्टी, वनस्पलत आवरर् और पानी जै से नि हुए प्राकृलतक संसाधनों का दोहन, संरिर् और लवकास करके
पाररक्तस्थनतक संतुलन को पु नस्थाानपत करना।
● मृदा अपवाह, मृदा हानन की रोकथाम।
● प्राकृनतक वनस्पनत का पु नजानन।
● वषाा जल संचयन और भू जि स्तर का पुनभथ रर्।
● बहु-फसिीय खे ती को सिम करना और लवलवध कृलि-आधाररत गलतलवलधयों की शुरूआत, जो जिग्रहर् िेत्
में रहने वािे िोगों को स्र्ायी आजीलवका प्रदान करने में मदद करती है ।
दे श की बारहमासी ननदयों को मौसमी ननदयों से जोडना एकीकृत जलसम्भर नवकास कायाक्रम का घटक
और लाभ नही ं है ।
स्रोत: संघ िोक सेवा आयोग, लसलवि सेवा प्रारं लभक परीिा- 2014

24. उत्तर: d
व्याख्या:
कथन 1 सही नही ं है : यह पररयोजना रोजगार महाननदे शालय, श्रम और रोजगार मंत्रालय द्वारा कायााक्तन्वत
की जा रही है । केंद्रीय श्म और रोजगार मंत्ािय ने अपने सार् पंजीकृत नौकरी चाहने वािों के लिए अपने रािरीय
कैररयर सेवा पररयोजना के माध्यम से मु फ्त ऑनिाइन कररयर कौशि प्रलशिर् भी शु रू लकया है ।
कथन 2 सही नही ं है : यह नौकरी चाहने वालों (नशनक्षत) और ननयोिाओं, प्रनशक्षण और कररयर मागा दशान
चाहने वाले उिीदवारों और प्रनशक्षण एवं कररयर परामशा प्रदान करने वाली एजेंनसयों के बीच अं तर को
पाटने की नदशा में काम करता है । इसका उद्दे श्य नौकरी नमलान, कररयर परामशा, व्यावसानयक
मागा दशान, कौशल नवकास पाठ्यक्रमों की जानकारी, इं टना नशप आनद जैसी नवनभन्न प्रकार की रोजगार
संबंधी सेवाएं प्रदान करना है । सभी सेवाएं नौकरी चाहने वािों, लनयोक्ताओं, प्रलशिर् प्रदाताओं सलहत सभी
लहतधारकों और िेसमें ट संगठन के लिए लनिः शुल्क हैं ।

25. उत्तर: c
व्याख्या:
● कथन 1 सही है : भारत सरकार द्वारा शु रू की गई सभी के लिए लकफायती एिईडी द्वारा उन्नत ज्योलत
(उजािा) योजना मु ख्य रूप से ररयायती दरों पर एलईडी बल्ब नवतररत करके ऊजाा दक्षता और संरक्षण
को बढावा दे ने पर केंनद्रत है । इस पहि का उद्दे श्य उद्दीप्त बल्ों को एिईडी से बदिना है , जो काफी कम
लबजिी की खपत करते हैं और िं बे समय तक चिते हैं , लजससे दे श भर में घरों और वालर्न्वज्यक प्रलतष्ठानों में
कुि ऊजाथ खपत कम हो जाती है ।
● कथन 2 सही नही ं है : उजािा योजना का प्रार्लमक उद्दे श्य सौर पैनिों के उत्पादन को बढाना नहीं है । जबलक
सौर ऊजाथ सलहत नवीकरर्ीय ऊजाथ स्रोतों को बढावा दे ना भारत की व्यापक ऊजाथ नीलत का एक लहस्सा है ,
उजािा योजना लवशेि रूप से एिईडी प्रकाश प्रौद्योलगकी को अपनाने के माध्यम से ऊजाथ दिता को िलित
करती है । सौर पैनि उत्पादन उद्योग को सीधे प्रभालवत करने के स्र्ान पर ऊजाथ की मां ग और ऊजाथ खपत
के पयाथ वरर्ीय प्रभाव को कम करने पर ध्यान केंलद्रत लकया गया है ।

26. उत्तर: c
व्याख्या:
● कथन 1 सही है : कायाकल्प और शहरी पररवतथन के लिए अटि लमशन (अमृत) 2015 में दे श भर के
चयननत 500 शहरों और कस्ों में शुरू नकया गया था।यह नमशन चयननत शहरों और कस्ों में जल
आपू नता, सीवरे ज और सेप्टेज प्रबंधन, तूफान जल ननकासी, हररत स्थान और पाका और गै र-मोटर
चानलत शहरी पररवहन के क्षे त्रों में आधारभूत संरचना के नवकास पर केंनद्रत है । इस लमशन में शहरी

For More Study Material, Visit: studyiq.com Page no.


9
Schemes Test #2

सुधार और िमता लनमाथ र् का एक समू ह शालमि लकया गया है । AMRUT नमशन को AMRUT 2.0 के
अं तगा त शानमल कर नदया गया है , नजसे 01 अिू बर, 2021 को शुरू नकया गया था। लमशन में गैर-
राजस्व जि में कमी, उपचाररत उपयोग लकए गए पानी का पुनचथक्रर्, जि लनकायों का कायाकल्प, दोहरी
प्रलवलि िेखा प्रर्ािी को बढाना, शहरी लनयोजन, शहरी लवि को मजबूत करना आलद के माध्यम से नागररकों
के जीवन को आसान बनाने पर एक सुधार एजें डा भी है ।
● कथन 2 सही नही ं है : भारत सरकार के शहरी नवकास मंत्रालय ने नवरासत शहरों के समग्र नवकास
पर ध्यान दे ने के साथ नवरासत शहर नवकास और संवधा न योजना (HRIDAY) शुरू की। हृदय का
मु ख्य उद्दे श्य लवरासत शहर की आत्मा के चररत् को संरलित करना और लनजी िेत् को शालमि करने सलहत
लवलभन्न तरीकों की खोज करके लवरासत से जुडे समावेशी शहरी लवकास को सुलवधाजनक बनाना है । हृदय
एक केंद्रीय क्षे त्र की योजना है , जहां 100% नवत्त पोषण भारत सरकार द्वारा प्रदान नकया जाएगा।
● कथन 3 सही है : भारत सरकार ने पयाटन मंत्रालय के तहत वषा 2014-2015 में प्रसाद योजना शुरू की।
प्रसाद योजना का पूर्थ रूप 'तीर्थ यात्ा कायाकल्प और आध्यान्वत्मक संवधथन अलभयान' है । यह योजना धानमाक
पयाटन अनु भव को समृद्ध करने के नलए पू रे भारत में तीथा स्थलों के नवकास और पहचान पर केंनद्रत
है । इसका उद्दे श्य संपूणा धानमाक पयाटन अनु भव प्रदान करने के नलए तीथा स्थलों को प्राथनमकता,
योजनाबद्ध और सतत तरीके से एकीकृत करना है । प्रसाद योजना का उद्दे श्य भारत में धालमथ क पयथटन के
लवकास और प्रचार का मागथ प्रशस्त करना है ।
स्रोत:
https://pib.gov.in/PressReleasePage.aspx?PRID=1885837 https://tourism.gov.in/schemes-
and-guidelines/schemes/scheme-guidelines-
prasad#:~:text=The%20full%20form%20of%20the,enriching%20the%20religious%20tourism
%20experience. https://vikaspedia.in/social-welfare/urban-poverty-alleviation-1/hriday-
scheme-and-guidelines

27. उत्तर: c
व्याख्या:
● कथन 1 और 3 सही हैं , जबनक कथन 2 सही नही ं है : भारतीय रे िवे ने भारत के लिए एक रािरीय रे ि
योजना (एनआरपी) - 2030 तैयार की है । योजना 2030 तक 'भलवष्य के लिए तैयार' रे िवे प्रर्ािी बनाने की
है । रािरीय रे ि योजना के प्रमु ख उद्दे श्य हैं :-
o माि ढु िाई में रे िवे की लहस्सेदारी को 45% तक बढाने के लिए पररचािन िमताओं और वालर्न्वज्यक
नीलत पहि दोनों के आधार पर रर्नीलत तैयार करना।
o मालगानडयों की औसत गनत 50 नकमी प्रनत घं टे तक बढाकर माल ढु लाई के समय को काफी हद
तक कम करना।
o नए समनपा त माल ढु लाई गनलयारों की पहचान करना।
o नए उच्च गनत रे ल गनलयारों की पहचान करना।
o यात्ी यातायात के लिए रोलिंग स्टॉक की आवश्यकता के सार्-सार् माि ढु िाई के लिए वैगन की
आवश्यकता का आकिन करना।
o 100% नवद् युतीकरण (हररत ऊजाा) और माल ढु लाई मॉडल नहस्से दारी बढाने के दोहरे उद्दे श्यों
को पू रा करने के नलए लोकोमोनटव आवश्यकता का आकलन करना।
o आवलधक लववरर् के सार् आवश्यक पूंजी में कुि लनवेश का आकिन करना।
o रोलिं ग स्टॉक के संचािन और स्वालमत्व, माि और यात्ी टलमथनिों के लवकास, टर ै क आधारभूत संरचना के
लवकास/संचािन आलद जै से िेत्ों में लनजी िे त् की लनरं तर भागीदारी।
स्रोत: https://pib.gov.in/PressReleasePage.aspx?PRID=1806617

28. उत्तर: b
व्याख्या:
● नवकल्प (a) सही है : आलर्थ क मामिों की कैलबने ट सलमलत ने हाि ही में प्रसार भारती अथाात ऑल इं नडया
रे नडयो (एआईआर) और दू रदशान (डीडी) के आधारभूत संरचना के नवकास के नलए केंद्रीय क्षे त्र

For More Study Material, Visit: studyiq.com Page no.


10
Schemes Test #2

योजना "प्रसारण अवसंरचना और ने टवका नवकास" (बीआईएनडी) के संबंध में सूचना और प्रसारण
मंत्रालय के प्रस्ताव को मंजूरी दे दी है । मं त्ािय की "प्रसारर् अवसंरचना और ने टवकथ लवकास" योजना
प्रसार भारती को उसके प्रसारर् अवसंरचना के लवस्तार और उन्नयन, सामग्री लवकास एवं संगठन से संबंलधत
लसलवि कायथ से संबंलधत खचों के लिए लविीय सहायता प्रदान करने का माध्यम है ।
● नवकल्प (b) सही नही ं है और नवकल्प (c) सही है : BIND योजना सावथजलनक प्रसारक को बेहतर बुलनयादी
ढां चे के सार् अपनी सुलवधाओं का अलधक उन्नयन करने में सिम बनाएगी, लजससे वामपंर्ी उग्रवाद (LWE),
सीमा और रर्नीलतक िे त्ों सलहत इसकी पहुं च का लवस्तार होगा और दशथ कों को उच्च गुर्विा वािी सामग्री
प्रदान की जाएगी। योजना का एक अन्य प्रमुि प्राथनमकता क्षे त्र घरे लू और अं तरााष्ट्रीय दोनों दशाकों के
नलए उच्च गु णवत्ता वाली सामग्री का नवकास करना और अलधक चैनिों को समायोलजत करने के लिए
डीटीएच िेटफॉमथ की िमता के उन्नयन के माध्यम से दशथ कों के लिए लवलवध सामग्री की उपिब्धता सुलनलित
करना है । बाहरी प्रसारण (ओबी) वै न की िरीद और टू नडयो को हाई डे नफननशन (एचडी) के नलए
तैयार करने के नलए उनका नडनजटल उन्नयन इस पररयोजना के एक नहस्से के रूप में नकया जाएगा।
● नवकल्प (d) सही है : वतथमान में , दू रदशथ न 28 िे त्ीय चैनिों सलहत 36 टीवी चैनि संचालित करता है और
ऑि इं लडया रे लडयो 500 से अलधक प्रसारर् केंद्र संचालित करता है । यह योजना दे श में एआईआर एफएम
टर ां समीटरों की कवरे ज को भौगोलिक िे त् के अनु सार 66% और जनसंख्या के अनु सार 80 % क्रमशिः 59%
और 68% तक बढाएगी। इस योजना में दू रदराज, आनदवासी, वामपं थी उग्रवाद और सीमावती क्षे त्रों में
रहने वाले लोगों को 8 लाि से अनधक डीडी फ्री नडश सेट टॉप बॉक्स (एसटीबी) के मुफ्त नवतरण की
भी पररकल्पना की गई है ।
स्रोत: https://www.pib.gov.in/PressReleasePage.aspx?PRID=1888540

29. उत्तर: c
व्याख्या:
जै सा लक नाम से पता चिता है , वायु खे ि में वायु के माध्यम से जु डी लवलभन्न खेि गलतलवलधयााँ शालमि हैं । इनमें
एयर-रे लसंग, एरोबेलटक्स, एयरो मॉडलिं ग, हैं ग ग्लाइलडं ग, पैराग्लाइलडं ग, पैरा मोटररं ग और स्काईडाइलवंग आलद जै से
खे ि शालमि हैं ।
● कथन 1 सही नही ं है : राष्ट्रीय वायु िेल नीनत 2022 (NASP 2022) नागररक उड्डयन मंत्रालय द्वारा
शुरू की गई है ।
● कथन 2 सही नही ं है : NASP 2022 भारत में एक सुरनक्षत, नकफायती, सुलभ, आनं ददायक और सतत
वायु िेल पाररक्तस्थनतकी तंत्र प्रदान करके 2030 तक भारत को शीषा िेल दे शों में से एक बनाने का
दृनष्ट्कोण प्रस्तुत करता है ।
● कथन 3 सही है : नई नीलत के तहत, भारत में वायु खे िों के लिए चार स्तरीय शासन संरचना होगी।
o एयर स्पोट्थ स फेडरे शन ऑफ इं लडया (ASFI) सवोच्च शासी लनकाय के रूप में
o व्यन्वक्तगत वायु खे िों या वायु खे िों के एक समू ह के लिए रािरीय संघ, जै सा उपयुक्त हो
o िे त्ीय (जै से पलिम/दलिर्/पूवोिर आलद) या रािरीय वायु खे ि संघों की राज्य और केंद्र शालसत प्रदे श स्तर
की इकाइयााँ , जै सा उपयुक्त हो; और
o लजिा-स्तरीय वायु खे ि संघ, जै सा उपयुक्त हो।
● कथन 4 सही है : NASP 2022 के मु ख्य उद्दे श्य:
o दे श में वायु खे ि संस्कृलत को बढावा दे ना
o वायु खे ि के आधारभू त संरचना, उपकरर्, संचािन, रखरखाव और प्रलशिर् सलहत सुरिा में अंतररािरीय
अच्छी प्रर्ाओं को अपनाने में सिम बनाना
o एक सरि, लहतधारक-अनु कूि और प्रभावी शासन संरचना लवकलसत करना
o वैलश्वक वायु खेि आयोजनों में भारतीय न्वखिालडयों की भागीदारी और सफिता बढाना; और
o आत्मननभार भारत अनभयान के अनु रूप भारत में वायु िेल उपकरणों के नडजाइन, नवकास और
नवननमााण को बढावा दे ना।
स्रोत: https://pib.gov.in/PressReleasePage.aspx?PRID=1831844

For More Study Material, Visit: studyiq.com Page no.


11
Schemes Test #2

30. उत्तर: c
व्याख्या:
"मैंग्रोव पाररक्तस्थनतकी तंत्र में सतत जलकृनष" (SAIME) पररयोजना का उद्दे श्य मैं ग्रोव की सुरिा और स्र्ानीय
जीवन न्वस्र्लतयों में सुधार पर ध्यान केंलद्रत करके जिीय कृलि प्रर्ालियों के सतत लडजाइन को बढावा दे ना है ।
वैलश्वक प्रकृलत लनलध (पररयोजना समियक) और ने चरिैं ड ईवी एक संघ में और दो चयलनत स्र्ानीय पररयोजना
भागीदारों के सार् लनकट सहयोग में एक सार् काम कर रहे हैं । इस पररयोजना को जमथन संघीय आलर्थ क सहयोग
और लवकास मं त्ािय (बीएमजेड) द्वारा लवि पोलित लकया गया है ।
● नवकल्प (c) सही है : पररयोजना उपायों का एक बडा लहस्सा सुंदरबन में कायााक्तन्वत नकया जा रहा है , जहां
दु लनया का सबसे बडा मैं ग्रोव वन भारत में संघीय राज्य पलिम बंगाि और बां ग्लादे श में खु िना लडवीजन में
10,000 लकमी² के िेत् में फैिा हुआ है । मैंग्रोव पाररक्तस्थनतकी तंत्र में सतत जलकृनष (SAIME) पहल के
तहत, नकसानों ने पनिम बंगाल के उत्तर 24 परगना के चैताल में और ननकटवती दनक्षण 24 परगना
के माधबपु र में 10 हे िेयर में मैंग्रोव पे डों के आसपास तालाबों में झी ंगा की िेती की है । दो स्र्ानीय
पररयोजना भागीदार बां ग्लादे श पयाथ वरर् और लवकास सोसायटी (BEDS) और भारतीय प्रकृलत पयाथ वरर् और
वन्यजीव सोसायटी (NEWS) हैं । पररयोजना का एजें डा मजबूत सहयोग और संचार संपकथ स्र्ालपत करना है ।
स्रोत:
https://www.naturland.de/en/producers/projects/international-commitment/sustainable-
aquaculture-in-mangrove.html

31. उत्तर: a
व्याख्या:
● कथन 1 सही नही ं है : आवास और शहरी मामिों के मं त्ािय ने फुटपाथ नवक्रेता (इसमें पारं पररक
कारीगरों का नबल्कुल भी उल्ले ि नही ं है ) को न केवल ऋण दे कर, बक्तल्क उनके समग्र नवकास और
आनथाक उत्थान के नलए सशि बनाने के नलए एक योजना प्रधानमंत्री फुटपाथ नवक्रेता आत्मननभार
नननध (प्रधानमंत्री स्वनननध) शुरू की। इस योजना का उद्दे श्य िगभग 50 िाख फुटपार् लवक्रेता को एक
साि के कायथकाि के लिए 10,000 रुपये तक के संपालश्वथ क मुक्त कायथशीि पूंजी ऋर् की सुलवधा प्रदान करना
है , तालक आसपास के पेरी-शहरी/ग्रामीर् िेत्ों सलहत शहरी िे त्ों में अपने व्यवसायों को लफर से शु रू करने
में मदद लमि सके।
● कथन 2 सही है : प्रधानमं त्ी स्वलनलध योजना इस प्रकार प्रोत्साहन प्रदान करती है :
o ऋण की ननयनमत पु नभुागतान पर 7% प्रनत वषा की दर से ब्याज सक्तिडी
o लनधाथ ररत लडलजटि िे नदे न करने पर प्रलत विथ 1200/- रुपये तक का कैशबैक
o ऋर् की बढी हुई अगिी लकश्त के लिए पात्ता
● कथन 3 सही नही ं है : आवास और शहरी मामिों के मं त्ािय (MoHUA) ने हाि ही में राज्यों/केंद्र शालसत
प्रदे शों और लवलभन्न केंद्रीय मं त्ाियों के वररष्ठ अलधकाररयों की उपन्वस्र्लत में 14 राज्यों/केंद्र शालसत प्रदे शों के
126 शहरों में 'स्वलनलध से समृ न्वि' कायथक्रम शुरू लकया है । कायाक्रम के तहत, भारत सरकार की 8
कल्याणकारी योजनाओं के नलए उनकी पात्रता का आकलन करने और पात्र योजनाओं की मंजूरी की
सुनवधा के नलए प्रधानमंत्री स्वनननध लाभानथायों और उनके पररवारों की सामालजक-आलर्थ क प्रोफाइलिं ग
की जाती है । इन योजनाओं में शालमि हैं :
o प्रधानमं त्ी जीवन ज्योलत बीमा योजना,
o प्रधानमं त्ी सुरिा बीमा योजना,
o प्रधानमं त्ी जन धन योजना,
o भवन और अन्य लनमाथ र् श्लमक (रोजगार और सेवा की शतों का लवलनयमन) अलधलनयम (बीओसीडब्ल्यू)
के तहत पंजीकरर्,
o प्रधानमं त्ी श्म योगी मानधन योजना,
o रािरीय खाद्य सुरिा अलधलनयम (एनएफएसए) पोटे लबलिटी िाभ - वन ने शन वन राशन काडथ
(ओएनओआरसी),
o जननी सुरिा योजना, एवं
o प्रधानमं त्ी मातृ वंदना योजना (PMMVY)।

For More Study Material, Visit: studyiq.com Page no.


12
Schemes Test #2

भारतीय गुर्विा पररिद (क्यूसीआई) इस कायथक्रम का कायाथ ियन भागीदार है ।


स्रोत:
https://pib.gov.in/PressReleaseIframePage.aspx?PRID=1816139
https://www.india.gov.in/spotlight/pm-street-vendors-atmanirbhar-nidhi-pm-svanidhi

32. उत्तर: c
व्याख्या:
● कथन 1 सही है : उदय (उज्ज्वि लडस्कॉम एश्योरें स योजना), लबजिी लवतरर् कंपलनयों (लडस्कॉम) के लविीय
बदिाव के लिए एक योजना है , इसका उद्दे श्य राज्य लडस्कॉम की पररचािन और लविीय दिता में सुधार
करना है ।
● कथन 2 सही है : यह योजना केवि राज्य के स्वालमत्व वािी DISCOMs पर िागू होगी। इस योजना के
प्रयोजन के लिए लडस्कॉम में संयुक्त उत्पादन, पारे िर् और लवतरर् उपक्रम शालमि हो सकते हैं ।
● कथन 3 सही है : योजना के उद्दे श्य हैं :
o DISCOMs का ऋर् बोझ कम करना।
o लडस्कॉम की पररचािन दिता में सुधार।
o उपभोक्ताओं को स्र्ायी लवद् युत आपूलतथ सुलनलित करना।
o नवीकरणीय ऊजाा के उपयोग को बढावा दे ना।
o लडस्कॉम की लविीय न्वस्र्लत को राज्य की लविीय न्वस्र्लत के सार् जोडकर उन पर लविीय अनु शासन िागू
करना।

33. उत्तर: d
व्याख्या:
 कथन 1 और 2 सही नही ं हैं : ग्रामीर् लवकास मं त्ािय (MoRD) ने भारत में हानशये पर क्तस्थत ग्रामीण
समुदायों से संबंनधत ग्रामीण मनहलाओं पर नवशेष ध्यान दे ने के सार्, "भारतीय ग्रामीण अथाव्यवस्था में
पररवतान" िाने के लिए, "प्रज्ज्विा चैिेंज" प्रारं भ करने की घोिर्ा की है । इसका उद्दे श्य ग्रामीण
अथाव्यवस्था को पररवनतात करने के नलए नए तथा अनभनव नवचार लाने हे तु रुलच एवं साझेदारी का
लवस्तार करना, लवशे िज्ञों, लशिालवदों, युवाओं, स्टाटथ -अप, स्वयं सहायता समू हों (SHG) आलद की शन्वक्त को
एकत् करना और उनका उपयोग करना है । यह भारत में ग्रामीर् आलर्थ क पररवतथन के लिए नए, अलभनव और
मापनीय मागथ प्रस्तु त करने के लिए, लवचारशीि मन्वस्तष्क को एक मं च प्रदान करने वािी अपनी प्रकार की
पहिी रािरव्यापी पहि है । इसका िक्ष्य भारत में ग्रामीर् अर्थव्यवस्र्ा में पररवतथन िाने के लिए, 5 नवोन्मेषी
नवचारों/मॉडलों का समथान करना है । इसे प्रधानमंत्री जनधन योजना के अं तगा त प्रारं भ नही ं नकया
गया है , बक्तल्क इसे ग्रामीण नवकास मंत्रालय द्वारा लॉन्च नकया गया है ।
स्रोत:
https://www.prajjwalachallenge.com/

34. उत्तर: c
व्याख्या:
 कथन 1 सही है : भारत सरकार द्वारा 5 जनवरी 2015 को स्टर ीट िाइलटं ग ने शनि प्रोग्राम (SLNP) प्रारं भ
लकया गया है , लजसमें पारं पररक स्टर ीट िाइट को स्माटथ तर्ा ऊजाथ दि िाइट एलमलटं ग डायोड (LED) स्टर ीट
िाइट से बदिने पर ध्यान केंलद्रत लकया गया है ।
o स्टर ीट िाइलटं ग ने शनि प्रोग्राम (SLNP) को लवद् युत मं त्ािय के अंतगथत, सावथजलनक िे त् के उपक्रमों
(PSU) की एक संयुक्त उद्यम कंपनी- एनजी एलफलशएं सी सलवथसेज लिलमटे ड (EESL) द्वारा कायाथ न्वित
लकया जा रहा है । इस कायथक्रम के अंतगथत, अब तक 28 राज्यों/केंद्र शालसत प्रदे शों को शालमि करते
हुए, 64 िाख से अलधक LED स्टर ीट िाइटें िगाई जा चुकी हैं।
 कथन 2 सही है : स्टर ीट िाइलटं ग ने शनि प्रोग्राम (SLNP) के लिए कोई लविीय आवंटन नहीं लकया जाता है ,
क्योंलक यह कायथक्रम भारत सरकार से लकसी बजटीय आवंटन के लबना संचालित लकया जाता है ।

For More Study Material, Visit: studyiq.com Page no.


13
Schemes Test #2

35. उत्तर: c
व्याख्या:
 कथन 1, 2 और 4 सही हैं : खाद्य सुरिा और मानक (पैकेलजं ग तर्ा िे बलिं ग) लवलनयम, 2011 के अनु सार,
लनमाथ ता के लिए मुख्य लेबल पर एनडनटव्स सनहत सामग्री की सूची, पोषण संबंधी जानकारी और
शाकाहारी/मांसाहारी जै सी सूचना दजथ करना अलनवायथ है ।
 कथन 3 सही नही ं है : लकसी भी एलजी प्रनतनक्रया की सं भावना के संबंध में नचनकत्सक द्वारा की गई
कोई भी नसफाररश, अलनवायथ नहीं है ।
स्रोत: संघ िोक सेवा आयोग, लसलवि सेवा प्रारं लभक परीिा- 2016

36. उत्तर: c
व्याख्या:
विथ 2022 तक पात् शहरी पररवारों को पक्का घर सुलनलित करके झुग्गीवालसयों सलहत आलर्थ क रूप से कमजोर
वगों/LIG और MIG श्े र्ी के बीच शहरी आवास की कमी (1.12 करोड घरों की मान्य मां ग) को संबोलधत करना।
(अब, इस योजना को 2024 तक बढा लदया गया है ) )
 कथन 1 सही है : प्रधानमं त्ी आवास योजना (शहरी) के अंतगथत पात्ता शतों में , लनम्नलिन्वखत शालमि हैं :
o िाभार्ी पररवार के नाम पर या उसके पररवार के लकसी भी सदस्य के नाम पर, भारत के लकसी भी लहस्से
में कोई पक्का मकान नहीं होना चालहए।
o वालिथ क आय सीमा
■ EWS (आलर्थ क रूप से कमजोर वगथ) के लिए- 3 िाख रुपये तक
■ LIG (लनम्न आय वगथ) के लिए- 3-6 िाख रुपये
■ MIG (मध्यम आय समू ह) के लिए - 6-18 िाख रुपये
 कथन 2 सही है : लाभाथी केवल एक घटक के अं तगा त लाभ प्राप्त कर सकते हैं ।
o EWS (आलर्थ क रूप से कमजोर वगथ) सभी घटकों के लिए पात् है ।
o LIG (लनम्न आय वगथ) और MIG (मध्यम आय समू ह) केवि क्रेलडट-लिंक्ड सन्विडी स्कीम (CLSS) के लिए
उपिब्ध है ।
o इस लमशन के अंतगथत केंद्रीय सहायता से लनलमथ त/अलधगृहीत आवास, पररवार की मलहिा मु न्वखया के नाम
पर या घर के पुरुि मु न्वखया और उसकी पत्नी के संयुक्त नाम पर होना चालहए और केवि उन मामिों में ,
जब पररवार में कोई वयस्क मलहिा सदस्य न हो, तो घर के पुरुि सदस्य के नाम पर मकान हो सकता
है ।
 कथन 3 सही है : सरकार ने प्रधानमं त्ी आवास योजना (PMAY) को बढावा दे ते हुए, लकफायती आवास िे त्
के लिए 'बुलनयादी ढां चे का दजाथ ' भी स्वीकृत लकया है ।

37. उत्तर: b
व्याख्या
 नवकल्प (a) सही नही ं है : प्रधान मंत्री सुरनक्षत मातृत्व अनभयान (PMSMA) 2016 में प्रारं भ लकया गया
र्ा। इस कायथक्रम का उद्दे श्य प्रत्येक माह के नौवें लदन सभी गभथ वती मलहिाओं को सावथभौलमक रूप से
लनिः शुल्क, सुलनलित, व्यापक तर्ा गुर्विापूर्थ प्रसवपूवथ दे खभाि प्रदान करना है । PMSMA, नननदा ष्ट्
सरकारी स्वास्थ्य सुनवधाओं पर गभाावस्था के दू सरे या तीसरे नतमाही में, मनहलाओं को प्रसवपूवा
दे िभाल सेवाओं के न्यू नतम पै केज की गारं टी प्रदान करता है ।
 नवकल्प (b) सही है : SUMAN योजना या सुरनक्षत मातृत्व आश्वासन योजना, केंद्रीय स्वास्थ्य एवं पररवार
कल्यार् मं त्ािय द्वारा प्रारं भ की गई एक मातृत्व िाभ पहि है । यह कायाक्रम गभावती मनहलाओं और
नवजात नशशुओ ं को नकफायती एवं गु णवत्तापू णा स्वास्थ्य दे िभाल समाधान प्रदान करता है । इस
योजना के अंतगथत गभथ वती मलहिाओं, बीमार नवजात लशशुओं और माताओं को प्रसव के बाद छह माह तक
शू न्य व्यय की सुलवधा प्राप्त होती है ।
 ● नवकल्प (c) सही नही ं है : LaQshya- लेबर रूम गु णवत्ता सुधार पहल, सरकारी मे लडकि कॉिे ज
अस्पतािों, लजिा अस्पतािों, उप-लजिा अस्पतािों तर्ा अन्य आवश्यक स्वास्थ्य सुनवधाओं में प्रसव कक्ष
और मातृत्व ऑपरे शन लर्एटरों में प्रदान की जाने वािी दे िभाल की गु णवत्ता में सुधार के नलए, केंद्रीय

For More Study Material, Visit: studyiq.com Page no.


14
Schemes Test #2

स्वास्थ्य एवं पररवार कल्याण मंत्रालय द्वारा प्रारं भ की गई एक योजना है ।


 ● नवकल्प (d) सही नही ं है : प्रधान मंत्री मातृ वं दना योजना- मनहला एवं बाल नवकास मंत्रालय द्वारा एक
मातृत्व लाभ कायाक्रम है , लजसमें 19 वषा या उससे अनधक उम्र की गभावती मनहलाओं को पहले जीनवत
जन्म के नलए ₹ 5000/- का नकद प्रोत्साहन लदया जाता है। यह प्रोत्साहन तीन लकस्तों में प्रदान लकया जाता
है और इसका दावा क्रमशिः 150 लदनों, 180 लदनों और बच्चे के जन्म के समय लकया जाता है ।
स्रोत:
India Yearbook - Chapter 14: Health and Family Welfare.
https://hrex.org/laqshya-scheme#:~:text=LaQshya-
%20Labour%2C%20quality%20improvement%20initiative%2C%20is%20a%20scheme,Hospit
als%2C%20Sub-
District%20Hospitals%2C%20and%20other%20essential%20health%20facilities.
https://www.myscheme.gov.in/schemes/pmmvy
Surakshit Matritva Aashwasan Yojana (myscheme.gov.in)

38. उत्तर: a
व्याख्या
● युग्म 1 सही सुमेनलत नही ं है : अटि पेंशन योजना (APY) भारत के नागररकों के लिए एक पेंशन योजना है , जो
असंगनित क्षे त्र के श्रनमकों पर केंनद्रत है । APY के अंतगथत, ग्राहकों के योगदान के आधार पर 60 विथ की आयु
पर प्रलत माह 1,000/- या 2,000/- या 3,000/- या 4,000 या 5,000/- रुपये की गारं टीकृत न्यू नतम पेंशन प्रदान
की जाएगी। आयकरदाता, APY के नलए पं जीकरण करने के पात्र नही ं हैं ।
● युग्म 2 सही सुमेनलत नही ं है : प्रधानमंत्री जीवन ज्योनत बीमा योजना (PMJJBY) लवि मं त्ािय की एक बीमा
योजना है , जो लकसी भी कारर् से मृ त्यु होने पर जीवन बीमा कवर प्रदान करती है । नकसी भी कारण से बीनमत
व्यक्ति की मृत्यु होने पर इस योजना के अं तगा त, जोक्तिम कवरे ज 2 लाि रुपये है ।

प्रधानमंत्री नकसान एक केंद्रीय क्षे त्र की योजना है , लजसमें सरकार से 100% नवत्त पोषण प्राप्त होता है । इस
योजना के अंतगथत, सभी भू लम धारक लकसान पररवारों को तीन समान लकश्तों में प्रलत विथ 6,000/- की आय
सहायता प्रदान की जाएगी।
● युग्म 3 सही सुमेनलत है : प्रधानमंत्री सुरक्षा बीमा योजना- एक दु घथटना बीमा योजना है , जो लकसी दु घथटना के
कारर् मृ त्यु या लवकिां गता के लिए आकन्वस्मक मृ त्यु एवं लवकिां गता कवर प्रदान करती है । इसमें प्रनत सदस्य
12/- रुपये प्रनत वषा का प्रीनमयम, खाताधारक के बैंक खाते से 'ऑटो डे लबट' सुलवधा के माध्यम से काटा जाएगा।
● युग्म 4 सही सुमेनलत नही ं है : नई पें शन योजना एक अन्य सेवालनवृलि योजना है , लजसमें लाभाथी सेवाननवृ नत्त
के बाद ननवे श की गई रानश का 60% ननकाल सकेंगे । इसे विथ 2004 में भारत सरकार द्वारा प्रस्तु त लकया गया
र्ा। मानसक पें शन प्राप्त करने के नलए शेष 40% को, वानषाकी में ननवे श करने की आवश्यकता होती है ।
NPS कोि का 60% कर-मुक्त है , जबलक शेष 40% कर योग्य है । पु रानी पें शन योजना, सरकारी कमथ चाररयों
को उनके अंलतम प्राप्त वेतन के आधार पर पेंशन प्रदान करती है । कमथ चारी को अंलतम आहररत वेतन का 50%
पेंशन के रूप में प्राप्त होता है । पें शन पर कोई टै क्स नही ं लगाया जाता है |
स्रोत:
https://www.india.gov.in/spotlight/atal-pension-yojana
https://www.myscheme.gov.in/schemes/pmjjby
https://www.myscheme.gov.in/schemes/pmsby
https://www.jagranjosh.com/general-knowledge/differences-between-old-pension-scheme-
and-new-pension-scheme-1663933333-1

39. उत्तर: c
व्याख्या:
● नवकल्प 1 सही है : अगिे पां च विों में आलदवासी समु दायों के 5000 युवाओं को लडलजटि रूप से कौशि प्रदान
करने एवं सशक्त बनाने के लिए, 15 मई 2020 को जनजातीय मामिों के मं त्ािय (MoTA) के सार् साझेदारी में

For More Study Material, Visit: studyiq.com Page no.


15
Schemes Test #2

फेसबुक द्वारा- गोइं ग ऑनिाइन ऐज िीडसथ (GOAL) कायथक्रम प्रारं भ लकया गया र्ा। इस पररयोजना का उद्दे श्य
सम्पू र्थ भारत में जनजातीय युवाओं को व्यन्वक्तगत रूप से सिाह दे ने के लिए लवलभन्न िेत्ों से संबंलधत 2500 प्रलसि
व्यन्वक्तयों की पहचान करना और उन्हें संगलठत करना है , लजसमें नीलत लनमाथ ता एवं प्रभावशािी व्यन्वक्त, लशिक,
किाकार, उद्यमी तर्ा सामालजक कायथकताथ शालमि हैं , जो अपने -अपने िे त्ों में अपनी उपिन्वब्धयों के लिए जाने
जाते हैं ।
● नवकल्प 2 सही नही ं है : संकल्प से लसन्वि योजना एक एकीकृत योजना है , लजसे केंद्र सरकार द्वारा रािर की
उन्नलत के लिए प्रारं भ लकया गया है । इस योजना के अंतगथत प्रमु ख सामालजक एवं आलर्थ क मु द्दों को शालमि लकया
जाएगा। इसमें दे श के 6-7 प्रमु ख िे त्ों या मु द्दों पर ध्यान केन्वन्द्त लकया जाएगा| इस योजना के अंतगथत लवलभन्न
आयोजनों के माध्यम से इन मु द्दों को उठाया जाएगा। ये मु द्दे हैं - स्वच्छ भारत, साक्षर भारत, गरीबी मुि,
भ्रष्ट्ाचार मुि, आतंकवाद मुि, साम्प्रदानयकता मुि और जानत-भेदभाव मुि भारत।
● नवकल्प 3 सही है : लघु वन उपज (MFP) के नलए न्यूनतम समथान मूल्य (MSP) तर्ा मू ल्य श्ृं खिा के
लवकास की योजना, जनजातीय मामिों के मं त्ािय (MoTA) द्वारा MFP संग्रहकताथ ओं को उलचत मू ल्य प्रदान करने ,
उनकी आय के स्तर में वृन्वि करने एवं MFP की सतत उपज सुलनलित करने के उद्दे श्य से, लवि विथ 2013-14 में
प्रारं भ की गई र्ी। िघु वन उपज (MFP) योजना के लिए न्यू नतम समर्थ न मू ल्य (MSP) का उद्दे श्य, संसाधन
आधार की संधारणीयता सु नननित करते हए जनजातीय संग्रहकतााओ,ं प्राथनमक प्रसंस्करण, भंडारण,
पररवहन आनद के नलए उनचत मूल्य सुनननित करने हे तु एक रूपरे िा स्थानपत करना है ।
● नवकल्प 4 सही है : वन धन योजना या वन धन स्कीम, 'न्यू नतम समर्थन मू ल्य (MSP) के माध्यम से िघु वन
उपज (MFP) के लवपर्न के लिए तंत् तर्ा MFP के लिए मू ल्य श्ृं खिा के लवकास' का एक घटक है , लजसे 14
अप्रैि 2018 को प्रारं भ लकया गया र्ा। रािरीय स्तर पर नोडि एजें सी के रूप में भारतीय जनजातीय सहकारी
नवपणन नवकास पररसंघ (TRIFED) द्वारा कायाथ न्वित वन धन स्टाटथ -अप, दे श की जनजातीय आबादी के
सामानजक-आनथाक नवकास के नलए एक सुनवचाररत माटर प्लान है ।
स्रोत:
https://pmjandhanyojana.co.in/sankalp-se-siddhi-scheme
https://trifed.tribal.gov.in/program/non-timber
https://trifed.tribal.gov.in/iipa
https://trifed.tribal.gov.in/pmvdy

40. उत्तर: d
व्याख्या
 MSME समाधान सूक्ष्म, िघु और मध्यम उद्यम मंत्ािय (MSME) का एक ऑनिाइन पोटथ ि है । यह MSME
को केंद्रीय मं त्ाियों/लवभागों/ CPSEs/राज्य सरकारों द्वारा नवलंनबत भुगतान के संबंध में, अपनी नशकायतें
ऑनलाइन दजा करने की अनु मनत प्रदान करता है ।
 MSME संबंध, केंद्र सरकार द्वारा MSME (सूक्ष्म, िघ तर्ा मध्यम उद्यम) के लिए प्रारं भ लकया गया एक
सावथजलनक खरीद पोटथ ि है । इसका िक्ष्य, केंद्रीय सावा जननक क्षे त्र के उद्यमों द्वारा MSEs (सूक्ष्म एवं लघु
उद्यम) से सावा जननक िरीद के कायाान्वयन की ननगरानी करना है ।
 MSME संपकथ पोटथ ि एक लडलजटि िेट़िॉमथ है , लजसमें रोज़गार के इच्छु क, 18 MSME प्रौद्योलगकी केंद्रों
के उिीर्थ प्रनशक्षु/छात्र तथा भतीकताा (लवलभन्न प्रलतलष्ठत रािरीय और बहुरािरीय कंपलनयां ) क्रमशिः रोजगार
प्राप्त करने और सही प्रकार की जनशन्वक्त प्राप्त करने के लिए स्वयं को पंजीकृत करते हैं ।

41. उत्तर: c
व्याख्या :
● कथन 1 सही नही ं है : भारत ने मोटे अनाजों की िपत और उत्पादन को प्रोत्सानहत करने के नलए एक
वै नश्वक पहल 'नमलेट इं टरने शनल इनननशएनटव फॉर ररसचा एं ड अवे यरने स' (एमआईआईआरए) शुरू
करने के नलए एक मसौदा प्रस्तुत नकया है । यह संयुक्त रािर द्वारा 2023 को अंतराथ िरीय मोटा अनाज विथ
घोलित करने के अनुरूप है , लजसका प्रस्ताव भारत द्वारा िाया गया र्ा और 72 दे शों ने इसका समर्थ न लकया
र्ा।

For More Study Material, Visit: studyiq.com Page no.


16
Schemes Test #2

● कथन 2 सही नही ं है : मीरा के शु रू होने के लिए, भारत "सीड मनी" का योगदान करे गा, जबलक प्रत्येक
G20 सदस्य को बाद में सदस्यता शु ल्क के रूप में अपने बजट में योगदान करना होगा। भारत मोटे अनाजों
का प्रमुि उत्पादक होने के कारण मीरा सनचवालय नदल्ली में होगा।
● कथन 3 सही है : मीरा का लक्ष्य इन फसलों पर अनु संधान का समथान करते हए दु ननया भर के मोटे
अनाजों से संबंनधत अनु संधान संगिनों को जोडना होगा। मोटे अनाज, छोटे दाने वािे अनाज होते हैं -
जै से सॉगथम (ज्वार), पिथ लमिे ट / मोती बाजरा (बाजरा), फॉक्सटे ि बाजरा (कां गनी / इटालियन बाजरा), छोटा
बाजरा ( कुटकी), कोदो बाजरा आलद। इन फसिों को चावि और गेहं की तुिना में बहुत कम पानी की
आवश्यकता होती है और ये मु ख्य रूप से विाथ आधाररत िे त्ों में उगाई जाती हैं ।
स्रोत : https:// Indianexpress.com/article/explained/miira-global-initiative-g20-miillets-india-
importance-8469052/

42. उत्तर: a
व्याख्या :
● कथन (a) सही है : आकृलत (AKRUTI) उन्नत ज्ञान और ग्रामीण प्रौद्योनगकी कायाान्वयन पहल का संलिप्त
रूप है । इस कायथक्रम के तहत, कई आकृलत (AKRUTI) समझौता ज्ञापनों पर हस्तािर लकए गए। महारािर
राज्य में तीन आकृलत (AKRUTI) नोड् स स्र्ालपत लकए गए र्े और महारािर सरकार द्वारा लवि पोिर् के सार्
चािू लकए गए र्े और अनु वती कारथ वाई के रूप में, अन्य राज्यों में अन्य गैर सरकारी संगठनों द्वारा स्व-
लविपोलित मोड में कुछ और नोड् स स्र्ालपत लकए गए र्े । आकृलत (AKRUTI) नोड् स ने गैर सरकारी संगठनों
के माध्यम से ग्रामीर् िे त् के लिए बाकथ प्रौद्योलगलकयों की उपयोलगता का प्रदशथ न लकया है , लजससे सामालजक
िाभ होता है । इसके अिावा, इसने प्रदलशथ त लकया है लक ग्रामीण क्षे त्र में तकनीकी रूप से उन्मुि मानव
संसाधन बाका वैज्ञाननकों और इं जीननयरों के मागा दशान में उनके उपयोग के नलए प्रौद्योनगनकयों को
तैनात करने में सक्षम है । इस कायाक्रम में बाकथ प्रौद्योनगनकयों पर आधाररत ग्रामीण तकनीकी-
उद्यनमता को प्रोत्सानहत करने की क्षमता है ।
स्रोत : इं लडया ईयर बुक 2023
https://www.barc.gov.in/akruti-tp/akrutitp.html

43. उत्तर: c
व्याख्या :
● कथन 1 सही है : केंद्रीय िे त् की योजनाएं मुख्य रूप से संघ सूची के नवषयों पर बनाई जाती हैं । केंद्रीय
क्षे त्र की योजनाएं केंद्र सरकार द्वारा कायााक्तन्वत की जाती हैं और योजना के लिए आवश्यक बजट या
लविपोिर् पूरी तरह से केंद्र सरकार द्वारा लकया जाता है।
o केंद्र प्रायोनजत योजनाएाँ - केंद्र प्रायोलजत योजनाएाँ - ये वे योजनाएाँ हैं , लजनमें केंद्र और राज्य दोनों की
लविीय भागीदारी होती है । राज्य की भागीदारी का अनु पात 50:50, 60:40, 70:30, 75:25, या 90:10 में
लभन्न हो सकता है ; लजसमें आमतौर पर केंद्र सरकार का योगदान अलधक होता है । केंद्र प्रायोनजत योजना
का कायाान्वयन राज्य/केंद्र शानसत प्रदे श सरकारों द्वारा नकया जाता है । केंद्र प्रायोलजत योजनाएं उन
िे त्ों पर बनाई जाती हैं जो राज्य सूची के अं तगा त आते हैं ।
● कथन 2 सही नही ं है : 2022 के बजट के अनु सार, 740 केंद्रीय क्षे त्र की योजनाएं और आवंटन हैं । पंद्रहवें
लवि आयोग ने अपनी ररपोटथ में केंद्र को अनावश्यक व्यय को कम करने के लिए बडी संख्या में केंद्रीय िेत्
की योजनाओं की समीिा करने और उन्हें तकथसंगत बनाने की सिाह दी। बजट दस्तावेज के अनु सार, सभी
मं त्ाियों में फैिी 130 केंद्र प्रायोनजत योजनाओं को 65 योजनाओं में "तकासंगत/संशोनधत" नकया गया
है ।
● कथन 3 सही है : महात्मा गां धी रािरीय ग्रामीर् रोजगार सृजन योजना एक केंद्र प्रायोलजत योजना है , जबलक
रािरीय ग्रामीर् स्वास्थ्य लमशन एक केंद्रीय िे त् की योजना है ।
स्रोत :
https://www.business-standard.com/about/what-is-centrally-sponsored-
schemes#:~:text=What%20is%20the%20difference%20between%20central%20sector%20sch
emes,centrally%20sponsored%20scheme%20is%20done%20by%20state%20governments.

For More Study Material, Visit: studyiq.com Page no.


17
Schemes Test #2

https://www.indianeconomy.net/splclassroom/what-is-the-difference-between-central-
sector-schemes-and-centrally-sponsored-schemes/

44. उत्तर: b
व्याख्या :
● कथन 1 सही है : पयाटन मंत्रालय के अं तगा त एक भारत श्रेष्ठ भारत की घोषणा 2015 में सरदार
वल्लभभाई पटे ल की 140वी ं जयंती के अवसर पर की गई थी। इसका उद्दे श्य नवनभन्न राज्यों और
केंद्रशानसत प्रदे शों के लोगों के बीच आपसी अं तः नक्रया और पारस्पररकता के माध्यम से सांस्कृनतक
नवनवधता का उत्सव मनाना है , तालक समझ की भावना पूरे दे श में गूंज सके। दे श के प्रत्येक राज्य और
केंद्रशालसत प्रदे श को भािा, सालहत्य, व्यं जन, त्योहारों, सां स्कृलतक कायथक्रमों, पयथटन आलद के िे त्ों में एक
दू सरे के सार् संरलचत जु डाव करने के लिए दू सरे राज्य/केंद्रशालसत प्रदे श के सार् जोडा गया है ।
● कथन 2 सही नही ं है : पयाटन मंत्रालय, अपनी चल रही प्रचार गनतनवनधयों के नहस्से के रूप में, भारत
के भीतर नवदे शी पयाटकों के आगमन और घरे लू यात्रा को बढाने के नलए भारत के नवनभन्न पयाटन
स्थलों और उत्पादों को बढावा दे ने के नलए 'अतुल्य भारत' ब्रांड नाम के तहत अं तरााष्ट्रीय और घरे लू
बाजारों में अनभयान जारी करता है ।
● कथन 3 सही है : पयाटन मंत्रालय ने दे श की समृद्ध नवरासत और संस्कृनत के बारे में नागररकों के बीच
जागरूकता पै दा करने और नागररकों को दे श के भीतर यात्रा करने के नलए प्रोत्सानहत करने के उद्दे श्य
से जनवरी 2020 में 'दे िो अपना दे श' पहल शुरू की, इस पहि के तहत, मं त्ािय ने दे खो अपना दे श के
समग्र लविय के तहत वेलबनार की एक श्ृं खिा आयोलजत की, लजसमें दे श की लवलवध संस्कृलत, लवरासत, स्र्िों
और पयथटन उत्पादों का प्रदशथन लकया गया।
● कथन 4 सही है : स्वदे श दशान योजना, पयाटन और संस्कृनत मंत्रालय, भारत सरकार द्वारा 2014-15 में
शु रू की गई एक केंद्रीय िे त् की योजना है , जो पयथटन अनुभव को समृ ि करने और रोजगार के अवसरों को
बढाने के लिए सभी लहतधारकों की जरूरतों और लचंताओं पर ध्यान केंलद्रत करने के प्रयासों को समन्वित
करके एकीकृत तरीके से उच्च पयथटक मू ल्य, प्रलतस्पधाथ और न्वस्र्रता के नवषय-आधाररत पयाटक सनकाट के
एकीकृत नवकास के नलए है । इसके तहत, नवकास के नलए 13 नवषयगत सनकाट् स- अर्ाथ त्िः उिर-पूवथ
भारत सलकथट, बौि सलकथट, लहमाियी सलकथट, तटीय सलकथट, कृष्णा सलकथट, रे लगस्तान सलकथट, जनजातीय
सलकथट, पयाथ वरर् सलकथट, वन्यजीव सलकथट, ग्रामीर् सलकथट, आध्यान्वत्मक सलकथट, रामायर् सलकथट और लवरासत
सलकथट की पहचान की गई है ।
स्रोत :
इं नडया ईयर बु क 2023
https://pib.gov.in/PressReleasePage.aspx?PRID=1844726
https://tourism.gov.in/swadesh-darshan-scheme

45. उत्तर: b
व्याख्या :
● युग्म 1 सही सुमेनलत है : भारत सरकार के स्वास्थ्य और पररवार कल्याण मंत्रालय ने राष्ट्रीय स्वास्थ्य
नमशन के तहत राष्ट्रीय बाल स्वास्थ्य कायाक्रम (आरबीएसके) लॉन्च नकया। इस कायाक्रम में जन्म से
18 वषा की आयु तक के बच्चों में 4 डी- नडफेि् स एट बथा (जन्म के समय दोष), नडनसज (रोग),
नडफीनसअं सी (कनमयां) और डे वलपमेंट नडले (नवकास में दे री), प्रारं नभक पहचान और तृतीयक स्तर
पर सजारी सनहत मुफ्त उपचार और प्रबं धन के नलए 32 सामान्य स्वास्थ्य क्तस्थनतयों की स्क्रीननंग शानमल
है ।
● युग्म 2 सही सुमेनलत नही ं है : स्वास्थ्य और पररवार कल्याण मंत्रालय ने 7 जनवरी 2014 को राष्ट्रीय
नकशोर स्वास्थ्य कायाक्रम (आरकेएसके) की शुरुआत की थी। इस कायथक्रम का उद्दे श्य 253 लमलियन
लकशोरों तक पहुं चना है , लजनमें िडके और िडलकयां , ग्रामीर् और शहरी, लववालहत और अलववालहत, स्कूि
में पढने वािे और स्कूि से बाहर के लकशोर शालमि हैं , लजनमें लवशे ि रूप से हालशए पर रहने वािे और वंलचत
समू हों पर ध्यान लदया जाएगा। यह 10-19 वषा की आयु वगा के नकशोरों के नलए स्वास्थ्य कायाक्रम है ।
यह कायथक्रम भारत में लकशोर स्वास्थ्य कायथक्रम के दायरे का लवस्तार करता है -यौन और प्रजनन स्वास्थ्य तक

For More Study Material, Visit: studyiq.com Page no.


18
Schemes Test #2

सीलमत होने से, अब इसमें पोिर्, चोट और लहं सा (लिं ग आधाररत लहं सा सलहत) गैर-संचारी रोग, मानलसक
स्वास्थ्य और मादक द्रव्यों का दु रुपयोग शालमि है । कायथक्रम की ताकत इसका स्वास्थ्य संवधथन दृलिकोर् है ।
● युग्म 3 सही सुमेनलत नही ं है : प्रधानमंत्री सुरनक्षत मातृत्व अनभयान भारत सरकार के स्वास्थ्य और
पररवार कल्याण मंत्रालय (एमओएचएफडब्ल्यू ) द्वारा शुरू नकया गया है । कायथक्रम का िक्ष्य हर महीने
की 9 तारीि को सभी गभावती मनहलाओं को सावा भौनमक रूप से ननः शुल्क, सुनननित, व्यापक और
गु णवत्तापू णा प्रसवपूवा दे िभाल प्रदान करना है ।
● युग्म 4 सही सुमेनलत है : "मदसथ एिोल्यू ट अफेक्शन" स्वास्थ्य और पररवार कल्यार् मं त्ािय का एक
रािरव्यापी कायथक्रम है जो स्तनपान को बढावा दे ने और स्वास्थ्य प्रर्ालियों के माध्यम से स्तनपान का समर्थन
करने के लिए परामशथ सेवाओं के प्रावधान पर लनरं तर ध्यान केंलद्रत करने के प्रयास में है । इसके तहत पहले
छह महीनों के नलए प्रारं नभक और नवशेष स्तनपान और उनचत नशशु और युवा बाल आहार
(आईवाईसीएफ) प्रथाओं को बढावा नदया जाता है ।
स्रोत :
https://pmsma.mohfw.gov.in/about-
scheme/#:~:text=What%20is%20the%20program%3F,the%209th%20of%20every%20month
https://vikaspedia.in/health/nrhm/national-health-programmes-1/rashtriya-kishor-swasthya-
karyakram-rksk
https://pib.gov.in/PressReleasePage.aspx?PRID=1809810

46. उत्तर: b
व्याख्या :
● कथन (b) सही है : नशीिे पदार्ों के लगरफ्तार अपरालधयों का अपनी तरह का पहिा डे टाबेस उपयोग के
लिए चािू कर लदया गया है । पोटा ल, ननदान या नगरफ्तार लकए गए नाको-अपरानधयों पर राष्ट्रीय एकीकृत
डे टाबे स, नारकोनटक्स कंटर ोल ब्यूरो (एनसीबी) द्वारा नवकनसत नकया गया है । यह नारकोलटक्स समिय
तंत् (एनसीओआरडी) पोटथ ि का लहस्सा है लजसे केंद्रीय गृह मं त्ी द्वारा शुरू लकया गया र्ा। ननदान िेट़िॉमथ
अपना डे टा आईसीजे एस (इं टर-ऑपरे बि आपरालधक न्यायलक प्रर्ािी) और ई-लप्रजन्स (एक क्लाउड-
आधाररत एन्विकेशन) ररपॉलजटरी (लनिे पागार) से प्राप्त करता है ।

स्रोत :
इं लडया ईयर बुक 2023
https://www.india.gov.in/website-national-integred-database-arrested-narco-offenders-
nidaan

47. उत्तर: d
व्याख्या :
● कथन 1 सही है : पीएम लवकास पारं पररक कारीगरों और लशल्पकारों को लविीय सहायता प्रदान करता है ।
यह समर्थ न लसफथ मौलद्रक नहीं है ; इसमें इन कारीगरों के पारं पररक कौशि को आधुलनक बनाने और बढाने
के लिए लडजाइन लकए गए कौशि लवकास कायथक्रम भी शालमि हैं । इसका उद्दे श्य इन पारं पररक लशल्पों को
प्रलतस्पधी और प्रासंलगक बनाना है , लजससे कारीगरों की आय में वृन्वि हो सके।
● कथन 2 सही है : योजना की पात्रता:
○ कोई भी श्लमक या कारीगर, जो असंगलठत िे त् में काम के औजारों और अपने हार्ों की मदद से
स्वरोजगार के आधार पर काम कर रहा हो या योजना में लदए गए 18 पररवार आधाररत पारं पररक
व्यवसायों में काम कर रहा हो, उसे लवश्वकमाथ योजना के लिए पात् माना जाएगा।
○ इस योजना के लिए केवि 18 विथ या उससे अलधक आयु का व्यन्वक्त ही आवेदन कर सकता है ।
○ कोई भी व्यन्वक्त जो इस योजना का िाभ िे ना चाहता है , उसे पंजीकरर् की लतलर् पर संबंलधत व्यवसाय
में संिग्न होना चालहए और पहिे से ही पीएमईजीपी, पीएम स्वलनलध और मु द्रा जै सी योजनाओं का 5 साि
से अलधक के लिए िाभार्ी नहीं होना चालहए ।

For More Study Material, Visit: studyiq.com Page no.


19
Schemes Test #2

पीएम लवश्वकमाथ योजना का िाभ एक पररवार का केवि एक ही सदस्य उठा सकता है । इसमें पररवार में

रहने वािे पलत, पत्नी और अलववालहत बच्चे शालमि हैं ।
○ लकसी भी सरकारी सेवा (केंद्र/राज्य) में कायथरत व्यन्वक्त और उस व्यन्वक्त के पररवार के सदस्य इस योजना
का िाभ नहीं उठा सकते हैं । इसमें पररवार में रहने वािे पलत-पत्नी और उनके अलववालहत बच्चे शालमि
हैं ।
● कथन 3 सही है : नवश्वकमाा योजना के अं तगा त आने वाले व्यापार
○ िकडी आधाररत
■ बढई (सुर्ार)
■ नाव बनाने वािा
○ िौह/धातु आधाररत/पत्थर आधाररत
■ शस्त्रसाज
■ िोहार
■ है मर और टू ि लकट लनमाथ ता
■ तािा बनाने वािा
■ लशल्पी (मू लतथकार, पत्थर तराशने वािा)
■ पत्थर तोडने वािा
○ सोना/चां दी आधाररत
■ सुनार
○ लमट्टी आधाररत
■ कुम्हार
○ चमडे पर आधाररत
■ मोची (चमथ कार)
■ जू ते बनाने वािा/जू ता कारीगर
○ वास्तु किा/लनमाथ र्
■ राजलमस्त्री
○ अन्य
■ गुलडया और न्वखिौना लनमाथ ता (पारं पररक)
■ नाई
■ मािा लनमाथ ता ( मािाकार )
■ धोबी
■ दजी
■ मछिी पकडने के जाि का लनमाथ ता

48. उत्तर: b
व्याख्या :
● कथन 1 सही है : मनरे गा एक केंद्र प्रायोनजत योजना है , जो प्रत्येक लविीय विथ में कम से कम 100 लदनों
के गारं टीकृत मजदू री रोजगार प्रदान करके दे श के ग्रामीर् िे त्ों में उन पररवारों की आजीलवका सुरिा को
बढाने का प्रयास करती है , लजनके वयस्क सदस्य अकुशि शारीररक काम करने के लिए स्वेच्छा से काम करते
हैं ।
● कथन 2 सही नही ं है : मनरे गा उन नजलों, जहां सौ प्रलतशत शहरी आबादी है , को छोडकर पू रे दे श को
शानमल करती है ।
● कथन 3 सही है : अलधलनयम का कायाथ ियन राज्य सरकारों द्वारा अलधलनयम के प्रावधानों के अनु सार उनके
द्वारा बनाई गई योजनाओं के अनु रूप लकया जाता है । मं त्ािय में प्राप्त सभी लशकायतें कानू न के अनु सार
उलचत कारथ वाई करने के लिए संबंलधत राज्यों को भे ज दी जाती हैं । गंभीर प्रकृलत की लशकायतों में , मं त्ािय
लशकायतों की जां च के लिए रािरीय स्तर के पयथवेिकों (एनएिएम) की लनयुन्वक्त करता है । सुधारात्मक कारथ वाई
करने के लिए एनएिएम की ररपोटथ संबंलधत राज्य सरकारों के सार् साझा की जाती है ।

For More Study Material, Visit: studyiq.com Page no.


20
Schemes Test #2

49. उत्तर: d
व्याख्या :
● कथन 1 सही है : राष्ट्रीय जल नमशन उन 8 राष्ट्रीय नमशनों में से एक है , जो जलवायु पररवतान पर राष्ट्रीय
काया योजना के आधार का लनमाथ र् करते हैं । रािरीय जि लमशन का उद्दे श्य "जि का संरिर् करना, अपव्यय
को कम करना तर्ा एकीकृत जि संसाधन लवकास और प्रबंधन के माध्यम से राज्यों में और राज्यों के भीतर
इसका समान लवतरर् सुलनलित करना" है ।
● कथन 2 सही नही ं है : राष्ट्रीय जल नीनत 1987 में जल सं साधन, नदी नवकास और गंगा संरक्षण नवभाग
द्वारा तैयार की गई थी, नजसकी बाद में वषा 2002 और 2012 में समीक्षा की गई और संशोधन नकया
गया। रािरीय जि नीलत को उलचत कारथ वाई के लिए सभी राज्यों/केंद्र शालसत प्रदे शों को भे ज लदया गया है।
उपिब्ध जानकारी के अनु सार, 16 राज्यों/केंद्रशालसत प्रदे शों ने अपनी राज्य जि नीलतयां बनाई और अपनाई
हैं । जि िे त् में वतथमान चुनौलतयों का समाधान करने के िक्ष्य के सार्, जि शन्वक्त मंत्ािय के जि संसाधन,
नदी लवकास और गंगा संरिर् लवभाग द्वारा रािरीय जि नीलत 2012 के संशोधन की पररकल्पना की गई है
और रािरीय जि नीलत को संशोलधत करने के लिए 5 नवंबर, 2019 को एक मसौदा सलमलत का गठन लकया
गया है ।
● कथन 3 सही है : जि शन्वक्त अलभयान एक समयबि, लमशन-मोड जि संरिर् अलभयान है । जि शन्वक्त
अलभयान दो चरर्ों में चिे गा: चरर् 1 सभी राज्यों और केंद्र शालसत प्रदे शों के लिए 1 जुिाई से 15 लसतंबर
2019 तक । और चरर् 2, 1 अक्टू बर से 30 नवंबर 2019 तक राज्यों और केंद्रशालसत प्रदे शों के लिए लजनमें
मानसून की वापसी होती है (आं ध्र प्रदे श, कनाथ टक, पुडुचेरी और तलमिनाडु )। जल शक्ति अनभयान : “कैच
द रे न” ( जेएसए : सीटीआर ) -2022, जेएसए की श्रृंिला में तीसरा, मानसून से पहिे और मानसून की
अवलध में "यह कहााँ लगरता है , जब लगरता है " लविय के सार् 29.3.2022 को 29 माचथ 2022 से 30 नवंबर
2022 के दौरान दे श भर के सभी लजिों (ग्रामीर् के सार्-सार् शहरी िे त्ों) के सभी ब्लॉकों को कवर करने
के लिए शु रू लकया गया है ।
● कथन 4 सही है : केंद्रीय भू जि बोडथ समय-समय पर कुओं की लनगरानी के एक ने टवकथ के माध्यम से िे त्ीय
स्तर पर पूरे भारत में भू जि स्तर की लनगरानी कर रहा है । दे श में भू जि लवकास और प्रबंधन के लवलनयमन
और लनयंत्र् के उद्दे श्य से 'पयाावरण (संरक्षण) अनधननयम, 1986' की धारा 3(3) के अं तगा त केंद्रीय
भूजल प्रानधकरण (सीजीडब्ल्यू ए) का गिन नकया गया है ।
स्रोत :
https://pib.gov.in/newsite/PrintRelease.aspx?relid=71513
https://pib.gov.in/PressReleasePage.aspx?PRID=1843395
https://pib.gov.in/PressReleasePage.aspx?PRID=1607166
https://pib.gov.in/PressReleasePage.aspx?PRID=1810544

50. उत्तर: c
व्याख्या :
● कथन 1 सही है : राष्ट्रीय स्वच्छ वायु कायाक्रम (एनसीएपी) को पयाावरण वन और जलवायु पररवतान
मंत्रालय (एमओईएफ एं ड सीसी) द्वारा 2019 में दे श भर में व्यापक तरीके से वायु प्रदू िर् की समस्या से
लनपटने के लिए एक दीघथकालिक, समयबि, रािरीय स्तर की रर्नीलत के रूप में शु रू लकया गया र्ा।
● कथन 2 सही नही ं है : इसका उद्दे श्य पूरे दे श में 2024 तक पालटथ कुिेट मैटर (लवलवक्त द्रव्य) (पीएम) सां द्रता
में 20 से 30% की कमी के िक्ष्य को प्राप्त करना है। 2026 तक एनसीएपी के अंतगथत आने वािे शहरों में
पालटथ कुिे ट मै टर सां द्रता में 40% की कमी का एक नया अद्यतन लक्ष्य ननधााररत नकया गया है ।
● कथन 3 सही है : एनसीएपी 124 गैर-प्रान्वप्त वािे शहरों (एनएसीएस) को शालमि करता है, लजसमें रािरीय
पररवेश वायु गुर्विा मानक (एनएएक्यूएस) िगातार 5 विों से अलधक है , सार् ही पंद्रहवें लवि आयोग-
लमलियन िस लसटीज चैिेंज फंड के अन्य 8 लमलियन से अलधक शहरों को शालमि लकया गया है । एनसीएपी
के तहत, शहर-नवनशष्ट् काया योजनाएं तैयार की गई हैं , नजनमें वायु गु णवत्ता ननगरानी ने टवका को
मजबू त करने , वाहनों और औद्योनगक उत्सजान को कम करने , सावा जननक जागरूकता बढाने आनद
के उपाय शानमल हैं ।
स्रोत :

For More Study Material, Visit: studyiq.com Page no.


21
Schemes Test #2

इं लडया ईयर बुक 2023


https://www.studyiq.com/articles/air-pollution-and-ncap/
स्रोत:
https://samadhaan.msme.gov.in/MyMsme/MSEFC/MSEFC_Welcome.aspx
https://www.financialexpress.com/industry/sme/msme-eodb-public-procurement-share-of-
goods-bought-from-msmes-hits-new-high-in-fy23-benefits-this-many-sellers/3019029/
http://sampark.msme.gov.in/?p=msmesampark

For More Study Material, Visit: studyiq.com Page no.


22

You might also like